Bullets for Nursing Basic Concepts Part 15

82
Bullets for Nursing Basic Concepts Part 15 If bleeding occurs after an injection, the nurse should apply pressure until the bleeding stops. If bruising occurs, the nurse should monitor the site for an enlarging hematoma. When providing hair and scalp care, the nurse should begin combing at the end of the hair and work toward the head. The frequency of patient hair care depends on the length and texture of the hair, the duration of hospitalization, and the patient’s condition. Proper function of a hearing aid requires careful handling during insertion and removal, regular cleaning of the ear piece to prevent wax buildup, and prompt replacement of dead batteries. The hearing aid that’s marked with a blue dot is for the left ear; the one with a red dot is for the right ear. A hearing aid shouldn’t be exposed to heat or humidity and shouldn’t be immersed in water. The nurse should instruct the patient to avoid using hair spray while wearing a hearing aid. The five branches of pharmacology are pharmacokinetics, pharmacodynamics, pharmacotherapeutics, toxicology, and pharmacognosy. The nurse should remove heel protectors every 8 hours to inspect the foot for signs of skin breakdown. Heat is applied to promote vasodilation, which reduces pain caused by inflammation. A sutured surgical incision is an example of healing by first intention (healing directly, without granulation). Healing by secondary intention (healing by granulation) is closure of the wound when granulation tissue fills the defect and allows reepithelialization to occur, beginning at the wound edges and continuing to the center, until the entire wound is covered.

Transcript of Bullets for Nursing Basic Concepts Part 15

Page 1: Bullets for Nursing Basic Concepts Part 15

Bullets for Nursing Basic Concepts Part 15

If bleeding occurs after an injection, the nurse should apply pressure until the bleeding stops. If bruising occurs, the nurse should monitor the site for an enlarging hematoma.

When providing hair and scalp care, the nurse should begin combing at the end of the hair and work toward the head.

The frequency of patient hair care depends on the length and texture of the hair, the duration of hospitalization, and the patient’s condition.

Proper function of a hearing aid requires careful handling during insertion and removal, regular cleaning of the ear piece to prevent wax buildup, and prompt replacement of dead batteries.

The hearing aid that’s marked with a blue dot is for the left ear; the one with a red dot is for the right ear.

A hearing aid shouldn’t be exposed to heat or humidity and shouldn’t be immersed in water.

The nurse should instruct the patient to avoid using hair spray while wearing a hearing aid.

The five branches of pharmacology are pharmacokinetics, pharmacodynamics, pharmacotherapeutics, toxicology, and pharmacognosy.

The nurse should remove heel protectors every 8 hours to inspect the foot for signs of skin breakdown.

Heat is applied to promote vasodilation, which reduces pain caused by inflammation.

A sutured surgical incision is an example of healing by first intention (healing directly, without granulation).

Healing by secondary intention (healing by granulation) is closure of the wound when granulation tissue fills the defect and allows reepithelialization to occur, beginning at the wound edges and continuing to the center, until the entire wound is covered.

Keloid formation is an abnormality in healing that’s characterized by overgrowth of scar tissue at the wound site.

The nurse should administer procaine penicillin by deep I.M. injection in the upper outer portion of the buttocks in the adult or in the midlateral thigh in the child. The nurse shouldn’t massage the injection site.

Page 2: Bullets for Nursing Basic Concepts Part 15

An ascending colostomy drains fluid feces. A descending colostomy drains solid fecal matter.

A folded towel (scrotal bridge) can provide scrotal support for thev patient with scrotal edema caused by vasectomy, epididymitis, or orchitis.

When giving an injection to a patient who has a bleeding disorder, the nurse should use a small-gauge needle and apply pressure to the site for 5 minutes after the injection.

Platelets are the smallest and most fragile formed element of the blood and are essential for coagulation.

To insert a nasogastric tube, the nurse instructs the patient to tilt the head back slightly and then inserts the tube. When the nurse feels the tube curving at the pharynx, the nurse should tell the patient to tilt the head forward to close the trachea and open the esophagus by swallowing. (Sips of water can facilitate this action.)

Families with loved ones in intensive care units report that their four most important needs are to have their questions answered honestly, to be assured that the best possible care is being provided, to know the patient’s prognosis, and to feel that there is hope of recovery.

Double-bind communication occurs when the verbal message contradictsv the nonverbal message and the receiver is unsure of which message to respond to.

Bullets for Nursing Basic Concepts Part 16

A nonjudgmental attitude displayed by a nurse shows that she neither approves nor disapproves of the patient.

Target symptoms are those that the patient finds most distressing.

A patient should be advised to take aspirin on an empty stomach, with a full glass of water, and should avoid acidic foods such as coffee, citrus fruits, and cola.

For every patient problem, there is a nursing diagnosis; for every nursing diagnosis, there is a goal; and for every goal, there are interventions designed to make the goal a reality.

Fidelity means loyalty and can be shown as a commitment to the profession of nursing and to the patient.

Administering an I.M. injection against the patient’s will and without legal authority is battery.

An example of a third-party payer is an insurance company.

Page 3: Bullets for Nursing Basic Concepts Part 15

The formula for calculating the drops per minute for an I.V. infusion is as follows: (volume to be infused × drip factor) ÷ time in minutes = drops/minute

On-call medication should be given within 5 minutes of the call.

Usually, the best method to determine a patient’s cultural or spiritual needs is to ask him.

An incident report or unusual occurrence report isn’t part of a patient’s record, but is an in-house document that’s used for the purpose of correcting the problem.

Critical pathways are a multidisciplinary guideline for patient care.

When prioritizing nursing diagnoses, the following hierarchy should be used: Problems associated with the airway, those concerning breathing, and those related to circulation.

The two nursing diagnoses that have the highest priority that the nurse can assign are Ineffective airway clearance and Ineffective breathing pattern.

A subjective sign that a sitz bath has been effective is the patient’s expression of decreased pain or discomfort.

For the nursing diagnosis Deficient diversional activity to be valid,the patient must state that he’s “bored,” that he has “nothing to do,” or words to that effect.

The most appropriate nursing diagnosis for an individual who doesn’t speak English is Impaired verbal communication related to inability to speak dominant language (English).

The family of a patient who has been diagnosed as hearing impaired should be instructed to face the individual when they speak to him.

Before instilling medication into the ear of a patient who is up to age 3, the nurse should pull the pinna down and back to straighten the eustachian tube.

To prevent injury to the cornea when administering eyedrops, the nurse should waste the first drop and instill the drug in the lower conjunctival sac.

Bullets for Nursing Basic Concepts Part 17

The keys to answering examination questions correctly are identifying the problem presented, formulating a goal for the problem, and selecting the intervention from the choices provided that will enable the patient to reach that goal.

After administering eye ointment, the nurse should twist the medication tube to detach the ointment.

Page 4: Bullets for Nursing Basic Concepts Part 15

When the nurse removes gloves and a mask, she should remove the gloves first. They are soiled and are likely to contain pathogens.

Crutches should be placed 6″ (15.2 cm) in front of the patient and 6″ to the side to form a tripod arrangement.

Listening is the most effective communication technique.

Before teaching any procedure to a patient, the nurse must assess the patient’s current knowledge and willingness to learn.

Process recording is a method of evaluating one’s communication effectiveness.

When feeding an elderly patient, the nurse should limit high-carbohydrate foods because of the risk of glucose intolerance.

When feeding an elderly patient, essential foods should be given first.

Passive range of motion maintains joint mobility. Resistive exercises increase muscle mass.

Isometric exercises are performed on an extremity that’s in a cast.

A back rub is an example of the gate-control theory of pain.

Anything that’s located below the waist is considered unsterile; a sterile field becomes unsterile when it comes in contact with any unsterile item; a sterile field must be monitored continuously; and a border of 1″ (2.5 cm) around a sterile field is considered unsterile.

A “shift to the left” is evident when the number of immature cells (bands) in the blood increases to fight an infection.

A “shift to the right” is evident when the number of mature cells in the blood increases, as seen in advanced liver disease and pernicious anemia.

Before administering preoperative medication, the nurse should ensure that an informed consent form has been signed and attached to the patient’s record.

A nurse should spend no more than 30 minutes per 8-hour shift providing care to a patient who has a radiation implant.

A nurse shouldn’t be assigned to care for more than one patient who has a radiation implant.

Page 5: Bullets for Nursing Basic Concepts Part 15

Long-handled forceps and a lead-lined container should be available in the room of a patient who has a radiation implant.

Usually, patients who have the same infection and are in strict isolation can share a room.

Bullets for Nursing Basic Concepts Part 18

Diseases that require strict isolation include chickenpox, diphtheria, and viral hemorrhagic fevers such as Marburg disease.

For the patient who abides by Jewish custom, milk and meat shouldn’t be served at the same meal.

Whether the patient can perform a procedure (psychomotor domain of learning) is a better indicator of the effectiveness of patient teaching than whether the patient can simply state the steps involved in the procedure (cognitive domain of learning).

According to Erik Erikson, developmental stages are trust versus mistrust (birth to 18 months), autonomy versus shame and doubt (18 months to age 3), initiative versus guilt (ages 3 to 5), industry versus inferiority (ages 5 to 12), identity versus identity diffusion (ages 12 to 18), intimacy versus isolation (ages 18 to 25), generativity versus stagnation (ages 25 to 60), and ego integrity versus despair (older than age 60).

When communicating with a hearing impaired patient, the nurse should face him.

An appropriate nursing intervention for the spouse of a patient who has a serious incapacitating disease is to help him to mobilize a support system.

Hyperpyrexia is extreme elevation in temperature above 106° F (41.1° C).

Milk is high in sodium and low in iron.

When a patient expresses concern about a health-related issue, before addressing the concern, the nurse should assess the patient’s level of knowledge.

The most effective way to reduce a fever is to administer an antipyretic, which lowers the temperature set point.

When a patient is ill, it’s essential for the members of his family to maintain communication about his health needs.

Ethnocentrism is the universal belief that one’s way of life is superior to others’.

When a nurse is communicating with a patient through an interpreter, the nurse should speak to the patient and the interpreter.

Page 6: Bullets for Nursing Basic Concepts Part 15

In accordance with the “hot-cold” system used by some Mexicans, Puerto Ricans, and other Hispanic and Latino groups, most foods, beverages, herbs, and drugs are described as “cold.”

Prejudice is a hostile attitude toward individuals of a particular group.

Discrimination is preferential treatment of individuals of a particular group. It’s usually discussed in a negative sense.

Increased gastric motility interferes with the absorption of oral drugs.

The three phases of the therapeutic relationship are orientation, working, and termination.

Patients often exhibit resistive and challenging behaviors in the orientation phase of the therapeutic relationship.

Bullets for Nursing Basic Concepts Part 19

Abdominal assessment is performed in the following order: inspection, auscultation, palpation, and percussion.

When measuring blood pressure in a neonate, the nurse should select a cuff that’s no less than one-half and no more than two-thirds the length of the extremity that’s used.

When administering a drug by Z-track, the nurse shouldn’t use the same needle that was used to draw the drug into the syringe because doing so could stain the skin.

Sites for intradermal injection include the inner arm, the upper chest, and on the back, under the scapula.

When evaluating whether an answer on an examination is correct, the nurse should consider whether the action that’s described promotes autonomy (independence), safety, self-esteem, and a sense of belonging.

Veracity is truth and is an essential component of a therapeutic relationship between a health care provider and his patient.

Beneficence is the duty to do no harm and the duty to do good. There’s an obligation in patient care to do no harm and an equal obligation to assist the patient.

Nonmaleficence is the duty to do no harm.

Frye’s ABCDE cascade provides a framework for prioritizing care by identifying the most important treatment concerns.

Page 7: Bullets for Nursing Basic Concepts Part 15

A = Airway. This category includes everything that affects a patentv airway, including a foreign object, fluid from an upper respiratory infection, and edema from trauma or an allergic reaction.

B = Breathing. This category includes everything that affects the breathing pattern, including hyperventilation or hypoventilation and abnormal breathing patterns, such as Korsakoff’s, Biot’s, or Cheyne-Stokes respiration.

C = Circulation. This category includes everything that affects the circulation, including fluid and electrolyte disturbances and disease processes that affect cardiac output.

D = Disease processes. If the patient has no problem with the airway, breathing, or circulation, then the nurse should evaluate the disease processes, giving priority to the disease process that poses the greatest immediate risk. For example, if a patient has terminal cancer and hypoglycemia, hypoglycemia is a more immediate concern.

E = Everything else. This category includes such issues as writing an incident report and completing the patient chart. When evaluating needs, this category is never the highest priority.

Rule utilitarianism is known as the “greatest good for the greatest number of people” theory.

Egalitarian theory emphasizes that equal access to goods and services must be provided to the less fortunate by an affluent society.

Active euthanasia is actively helping a person to die.

Brain death is irreversible cessation of all brain function.

Passive euthanasia is stopping the therapy that’s sustaining life.

A third-party payer is an insurance company.

Bullets for Nursing Basic Concepts Part 20

Utilization review is performed to determine whether the care provided to a patient was appropriate and cost-effective.

A value cohort is a group of people who experienced an out-of-the-ordinary event that shaped their values.

Voluntary euthanasia is actively helping a patient to die at the patient’s request.

Bananas, citrus fruits, and potatoes are good sources of potassium.

Page 8: Bullets for Nursing Basic Concepts Part 15

Good sources of magnesium include fish, nuts, and grains.

Beef, oysters, shrimp, scallops, spinach, beets, and greens are good sources of iron.

Intrathecal injection is administering a drug through the spine.

When a patient asks a question or makes a statement that’s emotionally charged, the nurse should respond to the emotion behind the statement or question rather than to what’s being said or asked.

The steps of the trajectory-nursing model are as follows:– Step 1: Identifying the trajectory phase– Step 2: Identifying the problems and establishing goals– Step 3: Establishing a plan to meet the goals– Step 4: Identifying factors that facilitate or hinder attainment of the goals– Step 5: Implementing interventions– Step 6: Evaluating the effectiveness of the interventions

A Hindu patient is likely to request a vegetarian diet.

Pain threshold, or pain sensation, is the initial point at which a patient feels pain.

The difference between acute pain and chronic pain is its duration.

Referred pain is pain that’s felt at a site other than its origin.

Alleviating pain by performing a back massage is consistent with the gate control theory.

Romberg’s test is a test for balance or gait.

Pain seems more intense at night because the patient isn’t distracted by daily activities.

Older patients commonly don’t report pain because of fear of treatment, lifestyle changes, or dependency.

No pork or pork products are allowed in a Muslim diet.

Two goals of Healthy People 2010 are: 1.  Help individuals of all ages to increase the quality of life and the number of years of optimal health 2. Eliminate health disparities among different segments of the population.

A community nurse is serving as a patient’s advocate if she tells av malnourished patient to go to a meal program at a local park.

Bullets for Nursing Basic Concepts Part 21

If a patient isn’t following his treatment plan, the nurse should first ask why.

Page 9: Bullets for Nursing Basic Concepts Part 15

Falls are the leading cause of injury in elderly people.

Primary prevention is true prevention. Examples are immunizations, weight control, and smoking cessation.

Secondary prevention is early detection. Examples include purified protein derivative (PPD), breast self-examination, testicular self-examination, and chest X-ray.

Tertiary prevention is treatment to prevent long-term complications.

A patient indicates that he’s coming to terms with having a chronic disease when he says, “I’m never going to get any better.”

On noticing religious artifacts and literature on a patient’s night stand, a culturally aware nurse would ask the patient the meaning of the items.

A Mexican patient may request the intervention of a curandero, or faith healer, who involves the family in healing the patient.

In an infant, the normal hemoglobin value is 12 g/dl.

The nitrogen balance estimates the difference between the intake and use of protein.

Most of the absorption of water occurs in the large intestine.

Most nutrients are absorbed in the small intestine.

When assessing a patient’s eating habits, the nurse should ask, “What have you eaten in the last 24 hours?”

A vegan diet should include an abundant supply of fiber.

A hypotonic enema softens the feces, distends the colon, and stimulates peristalsis.

First-morning urine provides the best sample to measure glucose, ketone, pH, and specific gravity values.

To induce sleep, the first step is to minimize environmental stimuli.

Before moving a patient, the nurse should assess the patient’s physical abilities and ability to understand instructions as well as the amount of strength required to move the patient.

To lose 1 lb (0.5 kg) in 1 week, the patient must decrease his weekly intake by 3,500 calories (approximately 500 calories daily). To lose 2 lb (1 kg) in 1 week, the patient

Page 10: Bullets for Nursing Basic Concepts Part 15

must decrease his weekly caloric intake by 7,000 calories (approximately 1,000 calories daily).

To avoid shearing force injury, a patient who is completely immobile is lifted on a sheet.

Bullets for Nursing Basic Concepts Part 22

To insert a catheter from the nose through the trachea for suction, the nurse should ask the patient to swallow.

Vitamin C is needed for collagen production.

Only the patient can describe his pain accurately.

Cutaneous stimulation creates the release of endorphins that block the transmission of pain stimuli.

Patient-controlled analgesia is a safe method to relieve acute pain caused by surgical incision, traumatic injury, labor and delivery, or cancer.

An Asian American or European American typically places distance between himself and others when communicating.

The patient who believes in a scientific, or biomedical approach to health is likely to expect a drug, treatment, or surgery to cure illness.

Chronic illnesses occur in very young as well as middle-aged and very old people.

The trajectory framework for chronic illness states that preferences about daily life activities affect treatment decisions.

Exacerbations of chronic disease usually cause the patient to seek treatment and may lead to hospitalization.

School health programs provide cost-effective health care for low-income families and those who have no health insurance.

Collegiality is the promotion of collaboration, development, and interdependence among members of a profession.

A change agent is an individual who recognizes a need for change or is selected to make a change within an established entity, such as a hospital.

The patients’ bill of rights was introduced by the American Hospital Association.

Page 11: Bullets for Nursing Basic Concepts Part 15

Abandonment is premature termination of treatment without the patient’s permission and without appropriate relief of symptoms.

Values clarification is a process that individuals use to prioritize their personal values.

Distributive justice is a principle that promotes equal treatment for all.

Milk and milk products, poultry, grains, and fish are good sources of phosphate.

The best way to prevent falls at night in an oriented, but restless, elderly patient is to raise the side rails.

By the end of the orientation phase, the patient should begin to trust the nurse.

Bullets for Nursing Basic Concepts Part 23

In a patient with hypokalemia (serum potassium level below 3.5 mEq/L), presenting signs and symptoms include muscle weakness and cardiac arrhythmias.

During cardiac arrest, if an I.V. route is unavailable, epinephrine can be administered endotracheally.

Pernicious anemia results from the failure to absorb vitamin B12 in the GI tract and causes primarily GI and neurologic signs and symptoms.

A patient who has a pressure ulcer should consume a high-protein, high-calorie diet, unless contraindicated.

The CK-MB isoenzyme level is used to assess tissue damage in myocardial infarction

After a 12-hour fast, the normal fasting blood glucose level is 80 to 120 mg/dl.

A patient who is experiencing digoxin toxicity may report nausea, vomiting, diplopia, blurred vision, light flashes, and yellow-green halos around images.

Anuria is daily urine output of less than 100 ml.

In remittent fever, the body temperature varies over a 24-hour period, but remains elevated.

Risk of a fat embolism is greatest in the first 48 hours after the fracture of a long bone. It’s manifested by respiratory distress.

To help venous blood return in a patient who is in shock, the nurse should elevate the patient’s legs no more than 45 degrees. This procedure is contraindicated in a patient with a head injury.

Page 12: Bullets for Nursing Basic Concepts Part 15

The pulse deficit is the difference between the apical and radial pulse rates, when taken simultaneously by two nurses.

To reduce the patient’s risk of vomiting and aspiration, the nurse should schedule postural drainage before meals or 2 to 4 hours after meals.

Blood pressure can be measured directly by intra-arterial insertion of a catheter connected to a pressure-monitoring device.

A positive Kernig’s sign, seen in meningitis, occurs when an attempt to flex the hip of a recumbent patient causes painful spasms of the hamstring muscle and resistance to further extension of the leg at the knee.

In a patient with a fractured, dislocated femur, treatment begins with reduction and immobilization of the affected leg

Herniated nucleus pulposus (intervertebral disk) most commonly occurs in the lumbar and lumbosacral regions.

Laminectomy is surgical removal of the herniated portion of an intervertebral disk.

Surgical treatment of a gastric ulcer includes severing the vagus nerve (vagotomy) to reduce the amount of gastric acid secreted by the gastric cells.

Valsalva’s maneuver is forced exhalation against a closed glottis, as when taking a deep breath, blowing air out, or bearing down.

Bullets for Nursing Basic Concepts Part 24

When mean arterial pressure falls below 60 mm Hg and systolic blood pressure falls below 80 mm Hg, vital organ perfusion is seriously compromised.

Lidocaine (Xylocaine) is the drug of choice for reducing premature ventricular contractions.

A patient is at greatest risk of dying during the first 24 to 48 hours after a myocardial infarction.

During a myocardial infarction, the left ventricle usually sustains the greatest damage.

The pain of a myocardial infarction results from myocardial ischemia caused by anoxia.

For a patient in cardiac arrest, the first priority is to establish an airway.

Page 13: Bullets for Nursing Basic Concepts Part 15

The universal sign for choking is clutching the hand to the throat.

For a patient who has heart failure or cardiogenic pulmonary edema, nursing interventions focus on decreasing venous return to the heart and increasing left ventricular output. These interventions include placing the patient in high Fowler’s position and administering oxygen, diuretics, and positive inotropic drugs as prescribed.

A positive tuberculin skin test is an induration of 10 mm or greater at the injection site.

The signs and symptoms of histoplasmosis, a chronic systemic fungal infection, resemble those of tuberculosis.

In burn victims, the leading cause of death is respiratory compromise. The second leading cause is infection.

The exocrine function of the pancreas is the secretion of enzymes used to digest carbohydrates, fats, and proteins.

A patient who has hepatitis A (infectious hepatitis) should consume a diet that’s moderately high in fat and high in carbohydrate and protein, and should eat the largest meal in the morning.

Esophageal balloon tamponade shouldn’t be inflated greater than 20 mm Hg.

Overproduction of prolactin by the pituitary gland can cause galactorrhea (excessive or abnormal lactation) and amenorrhea (absence of menstruation).

Intermittent claudication (pain during ambulation or other movement that’s relieved with rest) is a classic symptom of arterial insufficiency in the leg.

In bladder carcinoma, the most common finding is gross, painless hematuria.

Parenteral administration of heparin sodium is contraindicated in patients with renal or liver disease, GI bleeding, or recent surgery or trauma; in pregnant patients; and in women older than age 60.

Drugs that potentiate the effects of anticoagulants include aspirin, chloral hydrate, glucagon, anabolic steroids, and chloramphenicol.

For a burn patient, care priorities include maintaining a patent airway, preventing or correcting fluid and electrolyte imbalances, controlling pain, and preventing infection.

Page 14: Bullets for Nursing Basic Concepts Part 15

Medical and Surgical Nursing Review Questions 1

1. Which nursing intervention would be most appropriate for promoting the environmental safety of a client with a cognitive disorder?A. Applying an identification bracelet on the clientB. Maintaining daily routine care for the clientC. Placing a clock and a daily schedule in the client’s roomD. Using short sentences with simple words when speaking with the client

Correct Answer: ARationale: Applying an identification bracelet on the client would be most effective in helping to ensure environmental and client safety should the client wander. Other measures include installing alarms; instituting injury, fire, and poisoning precautions; providing adequate lighting; and keeping the bed in a low position. Maintaining a daily routine would be helpful for ensuring consistency and promoting optimal functioning. Clocks and daily schedules would be helpful for reorienting the client and promoting optimal cognitive function. Using short sentences with simple words would be appropriate for maximizing effective communication.

2. Which client complaint would lead the nurse to suspect premenstrual syndrome (PMS)?A. Fatigue and weight gain on the day prior to mensesB. Headache and mood swings occurring about 10 days prior to mensesC. Mood swings and breast tenderness with the onset of mensesD. Painful menstruation and large menstrual flow

Correct Answer: BRationale: Typically, PMS is manifested by complaints of headache, mood swings, irritability, weight gain, fatigue, and full, tender breasts, occurring approximately 10 days before menses in each cycle. Painful menstruation and a large menstrual flow are not associated with PMS.

3. When disposing of the plastic bags, tubing, syringes, and gloves used to administer antineoplastic drugs, the nurse should implement which nursing intervention?A. Avoiding contact with the equipment by allowing housekeeping to remove itB. Discarding all used equipment in a container marked “isolation”C. Disposing of all equipment in a container marked “bio-health hazard”D. Disposing of all used equipment in the regular trash receptacles

Correct Answer: CRationale: Any disposable equipment and supplies used for chemotherapy must be disposed of in a manner that protects the environment; placing the items in a container marked “bio-health hazard” is appropriate because these containers can be incinerated at a temperature of 2,200 to 2,500° F so that there is no residue. Only personnel trained in the proper handling of antineoplastic agents should handle the wastes. Infectious waste is incinerated at 1,700 to 1,800° F; residue is possible after incineration at these temperatures, making it an inappropriate method for the disposal of antineoplastic equipment and supplies. Because the equipment has been contaminated with material that is carcinogenic, special precautions are required.

Page 15: Bullets for Nursing Basic Concepts Part 15

4. Which assessment data for a client who is 1 day postabdominal surgery would warrant immediate nursing intervention?A. Blood pressure of 110/70 mm Hg and hematocrit of 42%B. Complaints of abdominal pain as anC. Hypoactive bowel sounds and a serum potassium of 3.7 mEq/LD. Rigid, hard, boardlike abdomen and a white blood cell (WBC) count of 20,000 mm

Correct Answer: DRationale: One day after abdominal surgery, the client’s abdomen should be soft, not rigid or hard. Also, the WBC count may be slightly elevated in response to the surgery, but an elevation of 20,000 mmis highly suggestive of an infectious process. A rigid, boardlike abdomen in conjunction with a seriously elevated WBC count suggests peritonitis and requires immediate intervention. The client’s blood pressure and hematocrit are within normal limits. One day after surgery, abdominal incisional pain would be expected and often is rated as high when using a scale from 1 to 10. The client’s hemoglobin level is within normal limits. Hypoactive bowel sounds would be expected 1 day after abdominal surgery. The client’s potassium level is within normal limits.

5. The nurse would include which nursing intervention for a client diagnosed with acute diverticulitis?A. Administration of stimulant laxativesB. Increased fluid intakeC. Continuation of client’s nothing-by-mouth statusD. High-fiber diet

Correct Answer: CRationale: During an acute episode of diverticulitis, measures focus on resting the colon, such as keeping the client on nothing-by-mouth status, administering I.V. fluids, and maintaining nasogastric suctioning and bedrest. Administering stimulant laxatives may be appropriate for restoring the client’s normal bowel elimination, but their use during an acute attack would only serve to irritate the bowel further. Increased fluid intake would be appropriate for diverticulosis. A high-fiber diet would be indicated for diverticulosis, but this type of diet would not be appropriate during an acute attack.

6. The nurse would include which nursing intervention in the care plan for a client with an L5-S1 intervertebral disc herniation?A. Assessing the skeletal traction insertion sites for infectionB. Encouraging the client to ambulate as much as possibleC. Positioning the client with his knees slightly flexed and the head of bed elevatedD. Preparing the client for lumbar puncture

Correct Answer: CRationale: Positioning the client with the head of the bed elevated and his knees slightly flexed increases the disc space and may help to decrease the client’s pain. Skeletal traction is not a treatment of choice for a herniated disc. The client with an intervertebral disc herniation should

Page 16: Bullets for Nursing Basic Concepts Part 15

be kept on bedrest. A lumbar puncture is not a diagnostic procedure for intervertebral disc herniation.

7. A 16-year-old client asks the nurse, “What caused me to have acne?” Which statement would be the nurse’s best response?A. “Acne is caused by an excess production of sebum.”B. “Acne is caused by not cleaning your face thoroughly every day.”C. “Eating lots of chocolate and candy causes you to have acne.”D. “The exact cause of acne is not really known.”

Correct Answer: DRationale: The exact cause of acne is not known, but evidence has shown that acne involves multiple factors, such as genetics, hormonal factors, and bacterial infections. Excess production of sebum results in seborrhea. Uncleanliness and dietary indiscretions, such as eating chocolate and candy, do not cause acne.

8. Which intervention would most important in the prevention of pressure ulcers?A. Applying external urine collection devicesB. Helping the client to maintain appropriate body positionC. Massaging reddened areas as soon as they are notedD. Turning the client every 2 hours

Correct Answer: DRationale: Turning the client frequently, such as every 2 hours, is one of the single most important interventions in preventing pressure ulcers because it helps to minimize the effects of pressure on the skin, allowing pressure to be redistributed with each turn. Applying an external urine collection device would be appropriate if the client is incontinent, but this action is not always relevant for every client and thus is not the most important. Helping the client to maintain appropriate body position is important, but it must be done in conjunction with frequent turning; maintaining body position without frequent turning would not be beneficial. Reddened areas should never be massaged because this increases tissue damage.

9. The client with a rectovaginal fistula is at high risk for infection. Which intervention would be the most important aspect of preventative nursing care?A. Administering antibioticsB. Ensuring adequate rest to enhance healingC. Monitoring temperature and white blood cell (WBC) countD. Performing perineal hygiene, including irrigations

Correct Answer: DRationale: The client with a rectovaginal fistula may experience fecal drainage via the vagina; preventing infection by keeping the vaginal area clean with irrigation, douches, and sitz baths would be most important. Administering antibiotics and ensuring adequate rest may be useful in promoting healing, but they are not preventative measures. Monitoring for symptoms of infection is important, but perineal hygiene is more effective as a preventative measure.

Page 17: Bullets for Nursing Basic Concepts Part 15

10. The client with a head injury is experiencing increased intracranial pressure (ICP). Which medication would the nurse anticipate administering?

A. Anticholinesterase agentsB. AnticonvulsantsC. Loop diureticsD. Osmotic diuretics

Correct Answer: DRationale: Osmotic diuretics such as mannitol are the preferred diuretic in the management of increased ICP to decrease cerebral edema and, therefore, decrease ICP. Anticholinesterase agents are used in the management of myasthenia gravis and are not helpful in decreasing ICP. Anticonvulsant medications would be used to treat seizure activity and are not helpful in decreasing ICP. Loop diuretics can be given in cases of increased ICP, but they are not a first-line agent.

FUNDAMENTALS

1. When removing a contaminated gown, the nurse should be careful that the first thing she touches is the:

1. Waist tie and neck tie at the back of the gown

2. Waist tie in front of the gown

3. Cuffs of the gown

4. Inside of the gown

Correct Answer: A. The back of the gown is considered clean, the front is contaminated. So, after removing gloves and washing hands, the nurse should untie the back of the gown; slowly move backward away from the gown, holding the inside of the gown and keeping the edges off the floor; turn and fold the gown inside out; discard it in a contaminated linen container; then wash her hands again.

2. Which of the following nursing interventions is considered the most effective form or universal precautions?

1. Cap all used needles before removing them from their syringes

2. Discard all used uncapped needles and syringes in an impenetrable protective container

3. Wear gloves when administering IM injections

Page 18: Bullets for Nursing Basic Concepts Part 15

4. Follow enteric precautions

Correct Answer: B. According to the Centers for Disease Control (CDC), blood-to-blood contact occurs most commonly when a health care worker attempts to cap a used needle. Therefore, used needles should never be recapped; instead they should be inserted in a specially designed puncture resistant, labeled container. Wearing gloves is not always necessary when administering an I.M. injection. Enteric precautions prevent the transfer of pathogens via feces.

3. All of the following measures are recommended to prevent pressure ulcers except: 1. Massaging the reddened are with lotion

2. Using a water or air mattress

3. Adhering to a schedule for positioning and turning

4. Providing meticulous skin care

Correct Answer: A. Nurses and other health care professionals previously believed that massaging a reddened area with lotion would promote venous return and reduce edema to the area. However, research has shown that massage only increases the likelihood of cellular ischemia and necrosis to the area.

4. Which of the following blood tests should be performed before a blood transfusion? 1. Prothrombin and coagulation time

2. Blood typing and cross-matching

3. Bleeding and clotting time

4. Complete blood count (CBC) and electrolyte levels.

Correct Answer: B. Before a blood transfusion is performed, the blood of the donor and recipient must be checked for compatibility. This is done by blood typing (a test that determines a person’s blood type) and cross-matching (a procedure that determines the compatibility of the donor’s and recipient’s blood after the blood types has been matched). If the blood specimens are incompatible, hemolysis and antigen-antibody reactions will occur.

5. The primary purpose of a platelet count is to evaluate the: 1. Potential for clot formation

2. Potential for bleeding

3. Presence of an antigen-antibody response

4. Presence of cardiac enzymes

Page 19: Bullets for Nursing Basic Concepts Part 15

Correct Answer: A. Platelets are disk-shaped cells that are essential for blood coagulation. A platelet count determines the number of thrombocytes in blood available for promoting hemostasis and assisting with blood coagulation after injury. It also is used to evaluate the patient’s potential for bleeding; however, this is not its primary purpose. The normal count ranges from 150,000 to 350,000/mm3. A count of 100,000/mm3 or less indicates a potential for bleeding; count of less than 20,000/mm3 is associated with spontaneous bleeding.

6. Which of the following white blood cell (WBC) counts clearly indicates leukocytosis? 1. 4,500/mm³

2. 7,000/mm³

3. 10,000/mm³

4. 25,000/mm³

Correct Answer:  D. Leukocytosis is any transient increase in the number of white blood cells (leukocytes) in the blood. Normal WBC counts range from 5,000 to 100,000/mm3. Thus, a count of 25,000/mm3 indicates leukocytosis.

7. After 5 days of diuretic therapy with 20mg of furosemide  (Lasix) daily, a patient begins to exhibit fatigue, muscle cramping and muscle weakness. These symptoms probably indicate that the patient is experiencing:

1. Hypokalemia

2. Hyperkalemia

3. Anorexia

4. Dysphagia

Correct Answer: A. Fatigue, muscle cramping, and muscle weaknesses are symptoms of hypokalemia (an inadequate potassium level), which is a potential side effect of diuretic therapy. The physician usually orders supplemental potassium to prevent hypokalemia in patients receiving diuretics. Anorexia is another symptom of hypokalemia. Dysphagia means difficulty swallowing.

8. Which of the following statements about chest X-ray is false? 1. No contradictions exist for this test

2. Before the procedure, the patient should remove all jewelry, metallic objects, and buttons above the waist

3. A signed consent is not required

4. Eating, drinking, and medications are allowed before this test

Page 20: Bullets for Nursing Basic Concepts Part 15

Correct Answer: A. Pregnancy or suspected pregnancy is the only contraindication for a chest X-ray. However, if a chest X-ray is necessary, the patient can wear a lead apron to protect the pelvic region from radiation. Jewelry, metallic objects, and buttons would interfere with the X-ray and thus should not be worn above the waist. A signed consent is not required because a chest X-ray is not an invasive examination. Eating, drinking and medications are allowed because the X-ray is of the chest, not the abdominal region.

9. The most appropriate time for the nurse to obtain a sputum specimen for culture is: 1. Early in the morning

2. After the patient eats a light breakfast

3. After aerosol therapy

4. After chest physiotherapy

Correct Answer:  A. Obtaining a sputum specimen early in this morning ensures an adequate supply of bacteria for culturing and decreases the risk of contamination from food or medication

10. A patient with no known allergies is to receive penicillin every 6 hours. When administering the medication, the nurse observes a fine rash on the patient’s skin. The most appropriate nursing action would be to:

1. Withhold the moderation and notify the physician

2. Administer the medication and notify the physician

3. Administer the medication with an antihistamine

4. Apply corn starch soaks to the rash

Correct Answer:  A. Initial sensitivity to penicillin is commonly manifested by a skin rash, even in individuals who have not been allergic to it previously. Because of the danger of anaphylactic shock, he nurse should withhold the drug and notify the physician, who may choose to substitute another drug. Administering an antihistamine is a dependent nursing intervention that requires a written physician’s order. Although applying corn starch to the rash may relieve discomfort, it is not the nurse’s top priority in such a potentially life-threatening situation.

Sample Nursing Board Exam Review Questions 3

1. All of the following nursing interventions are correct when using the Z-track method of drug injection except:

1. Prepare the injection site with alcohol

2. Use a needle that’s a least 1” long

Page 21: Bullets for Nursing Basic Concepts Part 15

3. Aspirate for blood before injection

4. Rub the site vigorously after the injection to promote absorption

Correct Answer:   D. The Z-track method is an I.M. injection technique in which the patient’s skin is pulled in such a way that the needle track is sealed off after the injection. This procedure seals medication deep into the muscle, thereby minimizing skin staining and irritation. Rubbing the injection site is contraindicated because it may cause the medication to extravasate into the skin.

2. The correct method for determining the vastus lateralis site for I.M. injection is to: 1. Locate the upper aspect of the upper outer quadrant of the buttock about 5 to 8 cm

below the iliac crest

2. Palpate the lower edge of the acromion process and the midpoint lateral aspect of the arm

3. Palpate a 1” circular area anterior to the umbilicus

4. Divide the area between the greater femoral trochanter and the lateral femoral condyle into thirds, and select the middle third on the anterior of the thigh

Correct Answer: D. The vastus lateralis, a long, thick muscle that extends the full length of the thigh, is viewed by many clinicians as the site of choice for I.M. injections because it has relatively few major nerves and blood vessels. The middle third of the muscle is recommended as the injection site. The patient can be in a supine or sitting position for an injection into this site.

3. The mid-deltoid injection site is seldom used for I.M. injections because it: 1. Can accommodate only 1 ml or less of medication

2. Bruises too easily

3. Can be used only when the patient is lying down

4. Does not readily parenteral medication

Correct Answer:  A. The mid-deltoid injection site can accommodate only 1 ml or less of medication because of its size and location (on the deltoid muscle of the arm, close to the brachial artery and radial nerve).

4. The appropriate needle size for insulin injection is: 1. 18G, 1 ½” long

2. 22G, 1” long

3. 22G, 1 ½” long

4. 25G, 5/8” long

Page 22: Bullets for Nursing Basic Concepts Part 15

Correct Answer:  D. A 25G, 5/8” needle is the recommended size for insulin injection because insulin is administered by the subcutaneous route. An 18G, 1 ½” needle is usually used for I.M. injections in children, typically in the vastus lateralis. A 22G, 1 ½” needle is usually used for adult I.M. injections, which are typically administered in the vastus lateralis or ventrogluteal site.

5. The appropriate needle gauge for intradermal injection is: 1. 20G

2. 22G

3. 25G

4. 26G

Correct Answer:  D. Because an intradermal injection does not penetrate deeply into the skin, a small-bore 25G needle is recommended. This type of injection is used primarily to administer antigens to evaluate reactions for allergy or sensitivity studies. A 20G needle is usually used for I.M. injections of oil-based medications; a 22G needle for I.M. injections; and a 25G needle, for I.M. injections; and a 25G needle, for subcutaneous insulin injections.

6. Parenteral penicillin can be administered as an: 1. IM injection or an IV solution

2. IV or an intradermal injection

3. Intradermal or subcutaneous injection

4. IM or a subcutaneous injection

Correct Answer:  A. Parenteral penicillin can be administered I.M. or added to a solution and given I.V. It cannot be administered subcutaneously or intradermally.

7. The physician orders gr 10 of aspirin for a patient. The equivalent dose in milligrams is: 1. 0.6 mg

2. 10 mg

3. 60 mg

4. 600 mg

Correct Answer: D. gr 10 x 60mg/gr 1 = 600 mg

8. The physician orders an IV solution of dextrose 5% in water at 100ml/hour. What would the flow rate be if the drop factor is 15 gtt = 1 ml?

1. 5 gtt/minute

Page 23: Bullets for Nursing Basic Concepts Part 15

2. 13 gtt/minute

3. 25 gtt/minute

4. 50 gtt/minute

Correct Answer:  C. 100ml/60 min X 15 gtt/ 1 ml = 25 gtt/minute

9. Which of the following is a sign or symptom of a hemolytic reaction to blood transfusion?

1. Hemoglobinuria

2. Chest pain

3. Urticaria

4. Distended neck veins

Correct Answer:  A. Hemoglobinuria, the abnormal presence of hemoglobin in the urine, indicates a hemolytic reaction (incompatibility of the donor’s and recipient’s blood). In this reaction, antibodies in the recipient’s plasma combine rapidly with donor RBC’s; the cells are hemolyzed in either circulatory or reticuloendothelial system. Hemolysis occurs more rapidly in ABO incompatibilities than in Rh incompatibilities. Chest pain and urticaria may be symptoms of impending anaphylaxis. Distended neck veins are an indication of hypervolemia

10. Which of the following conditions may require fluid restriction? 1. Fever

2. Chronic Obstructive Pulmonary Disease

3. Renal Failure

4. Dehydration

Correct Answer:  C. In real failure, the kidney loses their ability to effectively eliminate wastes and fluids. Because of this, limiting the patient’s intake of oral and I.V. fluids may be necessary. Fever, chronic obstructive pulmonary disease, and dehydration are conditions for which fluids should be encouraged.

Sample Nursing Board Exam Review Questions 4

1. All of the following are common signs and symptoms of phlebitis except: 1. Pain or discomfort at the IV insertion site

2. Edema and warmth at the IV insertion site

3. A red streak exiting the IV insertion site

Page 24: Bullets for Nursing Basic Concepts Part 15

4. Frank bleeding at the insertion site

Correct Answer: D. Phlebitis, the inflammation of a vein, can be caused by chemical irritants (I.V. solutions or medications), mechanical irritants (the needle or catheter used during venipuncture or cannulation), or a localized allergic reaction to the needle or catheter. Signs and symptoms of phlebitis include pain or discomfort, edema and heat at the I.V. insertion site, and a red streak going up the arm or leg from the I.V. insertion site.

2. The best way of determining whether a patient has learned to instill ear medication properly is for the nurse to:

1. Ask the patient if he/she has used ear drops before

2. Have the patient repeat the nurse’s instructions using her own words

3. Demonstrate the procedure to the patient and encourage to ask questions

4. Ask the patient to demonstrate the procedure

Correct Answer:  D. Return demonstration provides the most certain evidence for evaluating the effectiveness of patient teaching.

3. Which of the following types of medications can be administered via gastrostomy tube? 1. Any oral medications

2. Capsules whole contents are dissolve in water

3. Enteric-coated tablets that are thoroughly dissolved in water

4. Most tablets designed for oral use, except for extended-duration compounds

Correct Answer:   D. Capsules, enteric-coated tablets, and most extended duration or sustained release products should not be dissolved for use in a gastrostomy tube. They are pharmaceutically manufactured in these forms for valid reasons, and altering them destroys their purpose. The nurse should seek an alternate physician’s order when an ordered medication is inappropriate for delivery by tube.

4. A patient who develops hives after receiving an antibiotic is exhibiting drug: 1. Tolerance

2. Idiosyncrasy

3. Synergism

4. Allergy

Correct Answer:  D. A drug-allergy is an adverse reaction resulting from an immunologic response following a previous sensitizing exposure to the drug. The reaction can range from a rash or hives to anaphylactic shock. Tolerance to a drug

Page 25: Bullets for Nursing Basic Concepts Part 15

means that the patient experiences a decreasing physiologic response to repeated administration of the drug in the same dosage. Idiosyncrasy is an individual’s unique hypersensitivity to a drug, food, or other substance; it appears to be genetically determined. Synergism, is a drug interaction in which the sum of the drug’s combined effects is greater than that of their separate effects.

5. A patient has returned to his room after femoral arteriography. All of the following are appropriate nursing interventions except:

1. Assess femoral, popliteal, and pedal pulses every 15 minutes for 2 hours

2. Check the pressure dressing for sanguineous drainage

3. Assess a vital signs every 15 minutes for 2 hours

4. Order a hemoglobin and hematocrit count 1 hour after the arteriography

Correct Answer:  D. A hemoglobin and hematocrit count would be ordered by the physician if bleeding were suspected. The other answers are appropriate nursing interventions for a patient who has undergone femoral arteriography.

6. The nurse explains to a patient that a cough: 1. Is a protective response to clear the respiratory tract of irritants

2. Is primarily a voluntary action

3. Is induced by the administration of an antitussive drug

4. Can be inhibited by “splinting” the abdomen

Correct Answer: A. Coughing, a protective response that clears the respiratory tract of irritants, usually is involuntary; however it can be voluntary, as when a patient is taught to perform coughing exercises. An antitussive drug inhibits coughing. Splinting the abdomen supports the abdominal muscles when a patient coughs.

7. An infected patient has chills and begins shivering. The best nursing intervention is to: 1. Apply iced alcohol sponges

2. Provide increased cool liquids

3. Provide additional bedclothes

4. Provide increased ventilation

Correct Answer:  C. In an infected patient, shivering results from the body’s attempt to increase heat production and the production of neutrophils and phagocytotic action through increased skeletal muscle tension and contractions. Initial vasoconstriction may cause skin to feel cold to the touch. Applying additional bed clothes helps to equalize the body temperature and stop the chills. Attempts to cool the body result in further shivering, increased metabloism, and thus increased heat production.

Page 26: Bullets for Nursing Basic Concepts Part 15

8. A clinical nurse specialist is a nurse who has: 1. Been certified by the National League for Nursing

2. Received credentials from the Philippine Nurses’ Association

3. Graduated from an associate degree program and is a registered professional nurse

4. Completed a master’s degree in the prescribed clinical area and is a registered professional nurse.

Correct Answer:  D. A clinical nurse specialist must have completed a master’s degree in a clinical specialty and be a registered professional nurse. The National League of Nursing accredits educational programs in nursing and provides a testing service to evaluate student nursing competence but it does not certify nurses. The American Nurses Association identifies requirements for certification and offers examinations for certification in many areas of nursing., such as medical surgical nursing. These certification (credentialing) demonstrates that the nurse has the knowledge and the ability to provide high quality nursing care in the area of her certification. A graduate of an associate degree program is not a clinical nurse specialist: however, she is prepared to provide bed side nursing with a high degree of knowledge and skill. She must successfully complete the licensing examination to become a registered professional nurse.

9. The purpose of increasing urine acidity through dietary means is to: 1. Decrease burning sensations

2. Change the urine’s color

3. Change the urine’s concentration

4. Inhibit the growth of microorganisms

Correct Answer:  D. Microorganisms usually do not grow in an acidic environment.

10. Clay colored stools indicate: 1. Upper GI bleeding

2. Impending constipation

3. An effect of medication

4. Bile obstruction

Correct Answer:  D. Bile colors the stool brown. Any inflammation or obstruction that impairs bile flow will affect the stool pigment, yielding light, clay-colored stool. Upper GI bleeding results in black or tarry stool. Constipation is characterized by small, hard masses. Many medications and foods will discolor stool – for example, drugs containing iron turn stool black.; beets turn stool red.

Page 27: Bullets for Nursing Basic Concepts Part 15

Sample Review Questions on Medical and Surgical Nursing Part2

1. Which intervention would the nurse anticipate as the initial action to be included in the care plan for a client experiencing a tension pneumothorax?

A. Application of on occlusive petroleum dressingB. Increasing the ventilator’s tidal volumeC. Obtaining a chest X-rayD. Removal of an occlusive dressing

Correct Answer: DRationale: A tension pneumothorax occurs when the pressure increases in the pleural space. Thus, removing an occlusive dressing will release the increased pressure in the pleural space and help resolve the tension. Typically, the health care provider will insert a large bore needle initially and then a chest tube to aid in reinflating the lung. Applying an occlusive dressing will increase the pressure in the chest and worsen the tension pneumothorax. An occlusive dressing would be appropriate for an open pneumothorax. Increasing the tidal volume on the ventilator will increase the volume delivered to the chest, worsening the tension pneumothorax. The diagnosis of a tension pneumothorax is based on the client’s clinical presentation. It is a medical emergency that can quickly be fatal. Obtaining a chest X-ray wastes precious minutes that may permit the client to decompensate; it may be performed once the chest tube has been inserted and the initial build of pressure has been relieved.

2. When teaching a group of women about breast health awareness and breast self-examination (BSE) at a local community center, the nurse follows the American Cancer Society (ACS) recommendations. Which recommendation would the nurse include in the teaching program?

A. Bimonthly BSE and yearly mammograms beginning after the woman has had her first childB. Optional monthly BSE, yearly clinical examination, and yearly mammograms after age 40C. Quarterly BSE until the age of 70 after which breast health awareness is no longer necessaryD. Yearly BSE and follow up clinical examinations after onset of menses

Correct Answer: BRationale: The ACS recommends a yearly clinical examination and yearly mammograms in clients older than age 40. Monthly self-breast examination is an option for women starting in their 20s. The risk of breast cancer increases with age. At age 80, there is a 1 in 8 risk of developing breast cancer.

3. When providing postoperative care after a bowel resection to a client with a pre-existing history of chronic obstructive pulmonary disease (COPD) with frequent exacerbations, for which complication should the nurse be alert?

A. Acute respiratory failureB. Airway obstruction

Page 28: Bullets for Nursing Basic Concepts Part 15

C. AtelectasisD. Pneumothorax

Correct Answer: ARationale: The client is at high risk for developing acute respiratory failure because of his history of chronic lung disease requiring frequent intubations, the anesthesia used during surgery, and the experience of surgery. Airway obstruction and atelectasis are postoperative complications, but there is no evidence that this client would be at greater risk for these complication than anyone else. The operative procedure and the client’s medical history would not place this client at a greater risk for postoperative pneumothorax as compared to any other postoperative client.

4. The nurse is doing preoperative teaching for a client about to have a mechanical valve replacement. Which client statement indicates effective teaching?

A. “I need to make sure I have someone to care for me after this same-day surgery procedure.”B. “I will always need to take anticoagulants to prevent the formation of blood clots.”C. “I will need to take several days of steroids each time I have major dental work done.”D. “Because my valve is from a pig, I need to take precautions to prevent rejection of the valve.”

Correct Answer: BRationale: Following mechanical valve replacement surgery, clients need to be educated about the need for lifelong oral anticoagulant therapy. (Povine or bovine valve replacements do not require anticoagulants.) Valve replacement surgery is not performed as a day surgery procedure; it requires that the client be admitted to a critical care unit for constant monitoring due to the potential for complications. Prophylactic antibiotics, not steroids, are needed after valve replacement surgery. Rejection of the artificial valve is not a major problem associated with valve replacement surgery.

5. Which collaborative intervention would be included in the care plan for a client with a venous stasis ulcer to assist with healing?

A. Antiembolism stockingsB. Plaster cast sockC. Transcutaneous electrical nerve stimulator (TENS)D. Unna boot

Correct Answer: DRationale: An Unna boot is medicated gauze applied to the affected limb from the toes to the knees after the ulcer is cleaned. The boot is then wrapped in plastic wrap and hardens like a cast promoting venous return and preventing stasis. Antiembolism stockings are fit tightly and can traumatize an ulcer when applied. A plaster cast sock is usually applied to a residual limb following amputation to reduce edema. TENS is used as a pain relief measure; it would have no effect on healing.

Page 29: Bullets for Nursing Basic Concepts Part 15

6. A client with pulmonary edema is receiving mechanical ventilation with positive end-expiratory pressure (PEEP). When explaining to a student about the rationale for using PEEP, the nurse would indicate which rationale as its major purpose?

A. Allows the client to obtain needed restB. Increases pulmonary capillary pressureC. Improves area available for gas exchangeD. Increases the client’s carbon dioxide

Correct Answer: CRationale: PEEP helps keep the alveoli expanded, increasing the area available for gas exchange, thus improving the client’s oxygenation. PEEP has no effect on the client’s ability to rest, decreases pulmonary capillary pressure, and decreases the client’s carbon dioxide level by increasing the area for gas exchange.

7. The nurse teaches a client about residual limb care following an amputation and assesses that he understood the teaching when he demonstrates which behavior?

A. Applies lotions to keep the skin from crackingB. Elevates the residual limb on a pillow following surgeryC. Lies prone for several hours each dayD. Wraps the residual limb in adhesive bandages

Correct Answer: CRationale: Lying prone for several hours each day helps prevent hip contractures and demonstrates compliance with the treatment regimen. Using lotions keeps the skin soft; however, following an amputation, the skin needs to become tough. New guidelines recommend elevating the foot of the bed because a pillow can cause flexion contractures of the hip. Adhesive bandages irritate the skin, leading to sores, breakdown, and infection.

8. A client with a history of bigeminy who is on a lidocaine drip complains of light-headedness. Which intervention would the nurse implement

A. Calling the health care provider and getting a stat electrocardiogram (ECG)B. Checking the rhythm strip and assessing blood pressureC. Decreasing the lidocaine and instituting seizure precautionsD. Having the client lie down and administering atropine

Correct Answer: BRationale: Before doing anything else, the nurse needs to check the rhythm strip and assess the client’s blood pressure to determine the possible cause of the client’s complaints and gather additional data so that a full report can be made to the health care provider. An ECG is not needed for diagnosis of arrhythmia when a rhythm strip will suffice. The client is not exhibiting signs of lidocaine toxicity and, in fact, the lidocaine may need to be increased. Atropine is the drug of choice for sinus bradycardia, not premature ventricular contractions.

Page 30: Bullets for Nursing Basic Concepts Part 15

9. The nurse knows a client with chronic obstructive pulmonary disease (COPD) understands the discharge teaching when he makes which statement?

A. “I need to drink at least 2 liters of fluid every day.”B. “I need to take a sleeping pill every night so I wake up rested.”C. “I should do everything in the morning so I can rest later on.”D. “I should smoke only when I am not having difficulty breathing.”

Correct Answer: ARationale: Secretions are often very thick and difficult to expectorate for clients with COPD; drinking at least 2 liters of fluid per day will help to thin the secretions and aid in expectoration. Hypnotics and sedatives such as sleeping pills depress respirations and should be avoided. The client needs to pace himself and his activities to minimize energy expenditures and prevent exertion. The client should eliminate exposure to irritants such a smoking.

10. Which assessment finding indicates that furosemide (Lasix), a loop-diuretic, ordered for an elderly client is achieving its intended results?

A. +4 pitting edema in both legsB. Nontender calf muscles on palpationC. Relief of nocturnal leg crampingD. Systolic blood pressure of 150 mm Hg

Correct Answer: DRationale: Furosemide is commonly used as an initial step in treating hypertension. For the elderly client, a systolic blood pressure of 150 mm Hg would be considered normal and thus indicative that the drug therapy is effective. Pitting edema of +4 indicates that the drug is not achieving its intended result because fluid is still present; the client’s medication regime needs to be adjusted or changed. Furosemide has no effect on calf muscle; relief of tenderness in the calf is seen in deep vein thrombosis. Loop diuretics do not typically relieve cramping.

1. When caring for a client with arterial occlusive disease of the extremities, what would the nurse include in the client’s teaching plan?

A. Changing positions frequently and elevating the legs above the heart to promote venous return in the legsB. Elevating the arm on a pillow with the elbow higher than the shoulder and the hand higher than the elbowC. Elevating the foot of the bed about 6″ (15.2 cm) while the client is sleeping to promote venous returnD. Keeping the legs in a dependent position in relationship to the heart to improve peripheral blood flow

Correct Answer: DRationale: The client with arterial occlusive disease needs to enhance the blood supply to the body parts affected; keeping legs in a dependent position in relationship to the heart to improve

Page 31: Bullets for Nursing Basic Concepts Part 15

peripheral blood flow enhances the blood flow to the extremities. Changing positions frequently and elevating the legs above the heart to promote venous return in the legs should be included in teaching for the client with varicose veins. Elevating the arm on a pillow with the elbow higher than the shoulder and hand higher than the elbow helps to promote lymphatic drainage. Elevating the foot of the bed about 6″ while the client is sleeping to promote venous return is appropriate for the client with deep vein thrombosis.

2. While caring for a client with a new amputation, the dressing inadvertently comes off the stump. Which intervention should the nurse implement  first?

A. Bedside application of a large tourniquet to prevent massive hemorrhageB. Elevation of the limb above heart level to promote venous returnC. Maintenance of the client in a supine position to improve peripheral blood flowD. Immediate application of an elastic compression bandage wrapped around the limb

Correct Answer: DRationale: Because excessive edema will develop in a short time, resulting in delays in rehabilitation, the nurse should wrap the limb with an elastic compression bandage immediately. Before a tourniquet would be applied, the nurse would need to assess the client for signs and symptoms of bleeding because applying a tourniquet could compromise the circulatory and neurologic status of the limb. Elevating the limb above heart level could cause contractures; in this case, venous return is not a major concern. The supine position is contraindicated. The nurse needs to keep the stump elevated by raising the foot of the bed.

3. Which assessment finding would the nurse expect to assess in a client with emphysema?

A. Copious sputumB. Cor pulmonaleC. AnemiaD. Distant breath sounds

Correct Answer: DRationale: With emphysema, air trapping and chronic hyperexpansion of the lungs lead to distant breath sounds. Copious amounts of sputum are produced with chronic bronchitis; with emphysema, sputum production is usually scant. Cor pulmonale (right-sided heart failure) is more commonly associated with chronic bronchitis than emphysema. Polycythemia, an increase in red blood cells, may occur, but emphysema does not lead to anemia.

4. Following a thoracentesis, which assessment finding would warrant immediate intervention by the nurse?

A. Auscultation of crackles bilaterallyB. Complaints of pain at the needle insertion siteC. Prolonged periods of uncontrolled coughingD. Symmetrical respirations

Page 32: Bullets for Nursing Basic Concepts Part 15

Correct Answer: CRationale: Uncontrolled coughing in the client following a thoracentesis may indicate the development of pulmonary edema that requires immediate attention. Bilateral crackles may indicate underlying inflammation or congestion, but immediate attention is not necessary. Complaints of pain at the needle insertion site and symmetrical respirations are normal findings.

5. A client arrives in the emergency department following a motor vehicle accident with multiple injuries to the head, chest, and extremities with minimal bleeding. Which would the nurse assess first?

A. Airway statusB. Blood pressureC. Level of consciousnessD. Quality of peripheral pulses

Correct Answer: A

Rationale: When dealing with an emergency, the ABCs — airway, breathing, and circulation — are the priorities and must be maintained first. Blood pressure, neurological, and neurovascular assessments are important, but in this case, airway is the priority.

6. A client receiving nasogastric tube feedings for the past 48 hours develops a hacking cough, a fever of 100.6° F (38.1° C), and is moderately dyspneic. Which complication would the nurse suspect?

A. Aspiration pneumoniaB. Chronic obstructive pulmonary disease (COPD)C. Pleural effusionD. Pneumoconioses

Correct Answer: ARationale: Nasogastric tube feedings may result in aspiration leading to pneumonia, suggested by the hacking cough, low-grade fever, and moderate dyspnea. Clients with COPD have a chronic cough and usually are afebrile. Clients with pleural effusion usually have no cough and are afebrile. Clients with pneumoconioses present with chronic cough and progressive dyspnea.

7. A client is admitted to the health care facility with a diagnosis of acute arterial occlusion. While performing a physical assessment, what would the nurse expect to observe?

A. CrampingB. ElephatismC. Phantom painD. Pulselessness

Correct Answer: DRationale: Pulselessness is one of the common manifestations of acute arterial occlusion

Page 33: Bullets for Nursing Basic Concepts Part 15

secondary to cessation of blood flow distal to the occlusion. Cramping is a common complaint associated with varicose veins. Elephantism is an indication of secondary lymphedema. Phantom pain is pain noted following a limb amputation.

8. A client with leukemia is undergoing radiation therapy to the brain and spinal cord. In planning care for this client, the nurse would include which nursing intervention?

A. A scalp ointment to prevent dryness B. Avoiding washing off the target’s marksC. Not allowing the client to use a hat or scarfD. A dandruff shampoo twice daily

Correct Answer: BRationale: The marks made by the radiation oncologist guide the technician in configuring the external beam to irradiate the area in question without causing damage to other tissues. These marks must remain in place and should not be washed off. Ointments, which are petroleum-based, could cause a radiation burn to the area. The client should be encouraged to use a hat or scarf when in the sun to prevent damage to the scalp skin and at night to prevent loss of body heat through the scalp; hats and scarves also help to foster a positive body image. Dandruff shampoo includes harsh chemicals that could damage already fragile skin; the area being irradiated should be washed with water and the skin patted dry.

9. Which intervention would the nurse include in the teaching plan for a client diagnosed with gastroesophageal reflux disease (GERD)?

A. Avoiding eating within 2 hours of bedtimeB. Eating a high-fat, low-fiber dietC. Completing all antibioticsD. Sleeping with the head of the bed flat

Correct Answer: ARationale: Clients with GERD should avoid eating prior to retiring or lying down to decrease the incidence of reflux. The client with GERD will be prescribed a low-fat, high-fiber diet. Antibiotics are not used to treat GERD, although antibiotics are used for clients with <i>Helicobacter pylori</i> infection and peptic ulcer disease. The client with GERD should elevate the head on pillows or use blocks under the head of the bed to minimize reflux.

10. Which would the nurse include in the discharge teaching plan for an elderly client diagnosed with pneumonia?

A .Demonstration of postural drainage techniquesB. Demonstration of pursed lip breathingC. Discussion of proper use of oxygen therapyD. Instructions about increasing fluid intake

Correct Answer: DRationale: Pneumonia typically causes thick secretions that may be difficult for the elderly client

Page 34: Bullets for Nursing Basic Concepts Part 15

to expectorate; increasing fluid intake will help thin secretions, ultimately aiding in their removal. Postural drainage usually is recommended for clients diagnosed with bronchitis and emphysema. Pursed lip breathing and oxygen therapy usually are recommended for clients with chronic obstructive pulmonary disease. A client with pneumonia typically does not require oxygen at home.

1. A client who is complaining of right lower quadrant pain, nausea, and vomiting has a low-grade fever, rebound tenderness, and an elevated white blood cell (WBC) count. Which intervention should the nurse perform first?

A. Administering antacids for gastroenteritisB. Advising the client to assume a high Fowler’s position for a peptic ulcerC. Calling the surgeon in anticipation of an appendectomyD. Suggesting a course of antibiotics to treat peritonitis

Correct Answer: C

Rationale: The client is exhibiting classic findings associated with appendicitis, which requires surgery as soon as possible; notifying the surgeon should be the nurse’s first action. Rebound tenderness is not associated with gastroenteritis, which is characterized by generalized abdominal cramping, diarrhea, fever, and malaise. A high Fowler’s position would not alleviate pain produced by a peptic ulcer, which includes burning, aching, and gnawing pain. Nausea and vomiting are not generally associated with peritonitis, which is indicated by diffuse abdominal pain, rebound tenderness, fever, and an elevated WBC count.

2. Which assessment finding would be an appropriate indicator for evaluating a client with heart failure and a nursing diagnosis of decreased cardiac output?

A. Decreased intermittent claudicationB. Increased ability to walk to the bathroom without fatigueC. Increased heart rate by 10 beats per minuteD. Weight gain of 3 pounds in one day

Correct Answer: BRationale: Fatigue may be associated with decreased cardiac output; an increase in the client’s ability to ambulate to the bathroom without fatigue indicates improvement in cardiac output. A decrease in intermittent claudication indicates improved peripheral perfusion, but it does not demonstrate increased cardiac output. The body normally responds to a decrease in cardiac output by increasing the heart rate. Weight gain indicates fluid retention and a worsening of the client’s heart failure.

3. A client who has frostbite is complaining of pain. In addition to giving medication, which nursing intervention should the nurse implement?

A. Administration of sodium bicarbonateB. Elevation of the body part

Page 35: Bullets for Nursing Basic Concepts Part 15

C. Gentle massage of the affected areaD. Administration of warmed, humidified oxygen

Correct Answer: BRationale: Elevation of the body part helps to reduce the edema associated with frostbite. Sodium bicarbonate is indicated for the treatment of hypothermia. Massaging the affected area may result in further tissue damage. Warm, humidified oxygen is used as treatment for hypothermia.

4. A client scheduled for a biopsy of a mass asks the nurse to explain why this surgery is necessary. Which statement would be the nurse’s best response?

A. “The physician removes the precancerous mass to prevent cancer from occurring.”B. “This is diagnostic surgery done to confirm or rule out malignancy.”C. “This will provide a more realistic look to the body part.”D. “This will relieve your distress and help you to be more comfortable.”

Correct Answer: BRationale: A biopsy is performed to aid in diagnosing whether a mass is benign or malignant. Preventative surgery is done to remove tissue prior to its becoming cancerous; whether or not the mass is precancerous has yet to be determined. Reconstructive surgery provides a more realistic look to a body part. Palliative surgery is used to relieve the client’s distress and help make him more comfortable.

5. A client with deep venous thrombosis develops a sudden onset of severe leg pain. The limb becomes pale, cold, numb, and pulseless. What medical condition would the nurse suspect?

A. Acute arterial occlusionB. Dissecting aneurysmC. Postphlebitic syndromeD. Raynaud’s phenomenon

Correct Answer: ARationale: The change in color, temperature, sensation, and pulse accompanied by the sudden onset of pain (the classic “P’s” of assessment) all suggest an acute arterial occlusion. A dissecting aneurysm usually occurs in the chest, not the legs; a tearing or ripping sensation of pain in the anterior chest, back, epigastric region, or abdomen is common. Postphlebitic syndrome is characterized by a brownish discoloration of the skin, the hallmark sign. Raynaud’s phenomenon involves the episodic constriction of the small arteries or arterioles of the extremities, resulting in intermittent pallor and cyanosis of the skin, fingers, toes and, possibly, the ears or nose, followed by hyperemia, which may produce rubor.

6. When obtaining the history of a client admitted with endocarditis, which information from the client interview would the nurse consider as most significant?

Page 36: Bullets for Nursing Basic Concepts Part 15

A. Dental surgery in the recent pastB. History of coronary artery disease (CAD)C. History of marijuana useD. Prolonged use of steroid therapy

Correct Answer: ARationale: Dental surgery is one of the predisposing factors for the development of endocarditis because it may create a portal of entry for microorganisms. A history of valvular heart disease (not CAD), I.V. drug use (not marijuana use), and prolonged I.V. antibiotic therapy (not steroid therapy) are predisposing factors for endocarditis.

7. When assessing a client diagnosed with an abdominal aortic aneurysm, the nurse monitors the client for which signs and symptoms?

A. Intermittent episodes of high fever with chillsB. Paresthesias and loss of position senseC. Positive Homans’ sign and calf painD. Pulsatile mass and systolic bruit

Correct Answer:  DRationale: A pulsatile mass and systolic bruit are classic signs of an abdominal aortic aneurysm. Intermittent episodes of high fever with chills are associated with secondary lymphedema or other infections. Paresthesias and loss of position sense are associated with peripheral arterial occlusive disease as well as neurovascular and neurologic conditions. A positive Homans’ sign and calf pain are symptoms of deep vein thrombosis.

8. Which scientific rationale must the nurse keep in mind when administering oxygen to a client with chronic obstructive pulmonary disease (COPD)?

A. A facemask is necessary for delivery of adequateB. Oxygen is reserved for use when the client is short of breath.C. The client is encouraged to remove the oxygen as often as possible.D. The oxygen must be administered at a low rate.

Correct Answer: DRationale: The primary stimulus to breathe for the client with COPD is hypoxia. If oxygen were administered at too high a rate, the client’s respiratory drive would be depressed. The increased effectiveness of using a facemask as opposed to a nasal cannula has not been proven. Due to loss of supporting structures and narrowing of airways, the condition is irreversible; intermittent oxygen is not effective.

9. Which client would require the nurse to be on highest alert for the development of a pulmonary embolism (PE)?

A. A woman who has taken hormonal contraceptives for the past 2 yearsB. A client who has had laparoscopic gallbladder surgery

Page 37: Bullets for Nursing Basic Concepts Part 15

C. A client with arterial vascular disease and difficulty walkingD. A client who has experienced multiple trauma and fractures

Correct Answer: DRationale: A client with massive trauma and multiple orthopedic injuries is at increased risk for developing a PE. The injury may predispose the client to fat emboli and bony fragments that can become emboli, and the prolonged period of immobility that results from the injuries and their treatment further compounds the client’s risk. Women on hormonal contraceptives have a slightly higher risk for PE, but this risk is not as great as that for the client experiencing multiple trauma and fractures. The risk for cardiovascular complications increases after age 35 in women who smoke and after age 40 in women who do not smoke. Laparoscopic cholecystectomy is now considered a relatively minor procedure requiring a short hospitalization, usually in an outclient department. A client with arterial vascular disease may be at increased risk for pulmonary emboli but PE usually develops in the venous system.

10. Which assessment finding would be the most appropriate indicator for evaluating the adequacy of gas exchange for the postoperative client with a thoracotomy?

A. Effective coughing and deep-breathingB. Oxygen saturation level of 98%C. Report of breathing without difficultyD. Report of pain relief

Correct Answer: BRationale: Following a thoracotomy, the goal is to promote adequate gas exchange, evidenced by objective parameters including oxygen saturation, normal blood gases, and breath sounds. Effective coughing and deep breathing help to maintain a patent airway and promote lung expansion, but they do not ensure adequate gas exchange. Although client reports of breathing without difficulty are an important assessment, adequacy of gas exchange is best evaluated by objective findings. Assessment and pain relief is important, but pain relief is not a reliable indicator of adequate gas exchange.

1. When auscultating the breath sounds of a client with bacterial pneumonia, the nurse would expect to find which assessment data?

A. Adventitious breath sounds with crackles and wheezesB. Bronchial breath sounds over consolidated lung fieldsC. Decreased breath sounds with crackles and a pleural friction rubD. Wheezing with expiration more prolonged than inspiration

Correct Answer: BRationale: In normal, clear lungs, bronchial breath sounds would be heard over the large airways and vesicular breath sounds would be heard over the clear lungs. With pneumonia, exudate fills the air spaces producing consolidation and bronchial breath sounds over these

Page 38: Bullets for Nursing Basic Concepts Part 15

areas. Adventitious breath sounds, including crackles and wheezes, would be indicative of acute respiratory failure. Decreased breath sounds with crackles and a pleural friction rub would suggest a pulmonary embolism. Wheezing with expiration that is more prolonged than inspiration is indicative of chronic obstructive pulmonary disease.

2. When documenting the assessment finding of a client with emphysema who has an increase in the anteroposterior diameter of the chest, which term would the nurse use?

A. Barrel chestB. Flail chestC. Funnel chestD. Pigeon chest

Correct Answer: ARationale: Barrel chest is a term that refers to an increase in the anteroposterior diameter of the chest, resulting from overinflation of the lungs. A flail chest results from fractured ribs when a portion of the chest pulls inward upon inspiration. A funnel chest refers to a depression of the lower part of the sternum. A pigeon chest refers to an anterior displacement of the sternum protruding beyond the abdominal plane.

3. When caring for a client with a chest tube inserted in the right chest wall, which assessment data would lead the nurse to suspect that the client is experiencing a tension pneumothorax?

A. A cough with purulent sputumB. Frothy pink-tinged sputumC. Markedly decreased ventilation in the left lungD. Subcutaneous emphysema in the chest wall

Correct Answer: CRationale: Decreased ventilation in the opposite lung is indicative of a mediastinal shift, which leads to a tension pneumothorax. A cough with purulent sputum is usually seen in clients diagnosed with pneumonia. Hemoptysis is indicative of lung disease, such as pulmonary embolism and lung cancer. Subcutaneous emphysema, air accumulation in the tissues giving a crackling sensation when palpitated, is usually associated with chest trauma.

4. When evaluating risk for developing cancer, which client would the nurse identify as having the highest risk?

A. An asphalt road construction worker who eats meats and potatoesB. A new breast-feeding mother who works in a bankC. An oncology nurse who takes vitamins C and E dailyD. A vegetarian who works at a convenience store

Correct Answer: ARationale: Exposure to certain chemicals such as tar, soot, asphalt, oils, and sunlight put this occupation at the highest risk. Also, meats and potatoes are low in fiber, contributing to the risk

Page 39: Bullets for Nursing Basic Concepts Part 15

of cancer. Plus, some processed meats contain chemicals that have been implicated in the development of cancer. Breast-feeding does not increase the client’s risk of developing cancer. Office work also is not considered a risk factor. Working with cancer clients does not increase a person’s risk for developing cancer. Vitamins C and E have been shown to demonstrate preventative attributes. A vegetarian diet is considered to be a healthier diet for deduction of cancer risk because it provides increased fiber. Cruciferous vegetables have been shown to be preventative. Working in a convenience store does not increase risk.

5. A client with a history of coronary artery disease begins to experience chest pain. After putting the client on bedrest and administering a nitroglycerin tablet sublingually, which intervention should the nurse implement first?

A. Calling the health care providerB. Checking the heart’s creatine kinase MB (CK-MB) levelC. Getting a 12-lead electrocardiogram (ECG)D. Preparing the client for angioplasty

Correct Answer: CRationale: For the client experiencing chest pain, obtaining a 12-lead ECG is a priority to reveal possible changes occurring during an acute anginal attack that will be helpful in treatment. Before calling the health care provider, the nurse should obtain the results of the 12-lead ECG so that these results can be communicated to him. A CK-MB level may be ordered later and the client may need angioplasty in the near future, but getting the 12-lead ECG during the chest pain is the most important priority.

6. Which signs and symptoms would alert the nurse to the possibility of a major complication in a client with pericarditis?

A. Crushing chest pain and diaphoresisB. Dyspnea and copious blood-tinged, frothy sputumC. Hypotension and muffled heart soundsD. Tachycardia and oliguria

Correct Answer: CRationale: A major complication associated with pericarditis is pericardial effusion or cardiac tamponade manifested by hypotension and muffled heart sounds. Crushing chest pain and diaphoresis are signs of myocardial infarction. Dyspnea and copious blood-tinged, frothy sputum are signs of acute pulmonary edema, a complication of left-sided heart failure. Tachycardia and oliguria are signs of hemorrhagic shock.

7. Which assessment finding would the nurse identify as indicative of a client’s altered peripheral vascular function?

A. Ankle arm index pressure of 0.4B. Capillary refill time of less than 3 seconds

Page 40: Bullets for Nursing Basic Concepts Part 15

C. Diastolic blood pressure of 84 mm HgD. Pulses graded as being +4

Correct Answer: ARationale: The ankle arm index is an objective indicator of arterial disease. Normal value is 1.0. Values less than 0.5 indicate ischemic rest pain. A capillary refill time of less than 3 seconds is considered normal. A diastolic blood pressure of 84 mm Hg is considered within the normal range. Pulses graded as +4 are considered normal.

8. Which valvular disorder would the nurse suspect in a client presenting with fatigue, hemoptysis, and dyspnea on exertion?

A. Aortic insufficiencyB. Aortic stenosisC. Mitral insufficiencyD. Mitral stenosis

Correct Answer: DRationale: Mitral stenosis is an obstruction of blood flowing from the left atrium into the left ventricle, commonly manifested by progressive fatigue due to low cardiac output, hemoptysis, and dyspnea on exertion secondary to pulmonary venous hypertension. Aortic insufficiency refers to the backflow of blood from the aorta into the left ventricle during diastole; most clients are asymptomatic, except for a complaint of a forceful heartbeat. Aortic stenosis refers to a narrowing of the orifice between the left ventricle and the aorta; many clients experience no symptoms early on, but eventually develop exertional dyspnea, dizziness, and fainting. Mitral insufficiency refers to the backflow of blood from the left ventricle and aorta; many clients experience no symptoms early on, but eventually develop exertional dyspnea, dizziness, and fainting.

9. When developing a teaching plan for clients with chronic obstructive pulmonary disease (COPD) about the prevention of acute exacerbations, which topic should be included?

A. Administration of antibioticsB. Administration of oxygen as neededC. Performance of deep-breathing and coughing exercisesD. Elimination of exposure to pulmonary irritants

Correct Answer: DRationale: One aspect of exacerbation prevention focuses on eliminating the causes and contributory factors associated with COPD, such as pulmonary irritants (e.g., smoke, air pollution, occupational irritants, and allergies). Prevention would focus on eliminating these irritants. Antibiotics are used to treat bronchial infection during exacerbations, but they are not used prophylactically. Although oxygen is used in managing acute exacerbations, it is not a preventative measure. Coughing and deep breathing may help clients clear their airways and prevent further atelectasis, but they will not prevent exacerbation.

Page 41: Bullets for Nursing Basic Concepts Part 15

10. Which medication would the nurse expect the health care provider to order immediately for a client who is newly diagnosed with chronic obstructive pulmonary disease (COPD)?

A. A bronchodilatorB. A corticosteroidC. An anticoagulantD. An antitussive agent

Correct Answer: ARationale: Initially, for the client newly diagnosed with COPD, the health care provider would order a bronchodilator to open the airways and ease dyspnea. Corticosteroids may be ordered for the client with COPD, but they are usually used for acute exacerbations, not as an initial drug. Anticoagulants interfere with the clotting cascade and would be ordered for a client with an embolic disorder such as pulmonary embolism. An antitussive agent would be used for the client with coughing, such as that occurring with pneumonia.

Sample Nursing Board Exam Review Questions 1

1. Cherry carl is scheduled to have a hysterosalpingogram.  Which of the following instruction would you give her regarding this procedure?

1. She may feel some mild cramping when the dye is inserted

2. The sonogram of the uterus will reveal any tumors present

3. She will not able to conceive 3 months after the procedure

4. Many women experience mild bleeding as an after effect

Correct Answer: 1. She may feel some mild cramping when the dye is inserted

2.  Bob Carl asks you what artificial insemination by donor entails.  Which would be best answer?

1. Artificial sperm are injected vaginallyt to testtubal patency2. Donor sperm are introduced vaginally into the uterus or cervix

3. The husband’s sperm is administered intravenously weekly

4. Donor sperm are injected intraabdominally into each ovary

Correct Answer: 2. Donor sperm are introduced vaginally into the uterus or cervix

3.   Cheryl Carl is having a GIFT procedure.  What makes her a good  candidate for this procedure?

1. She has patent fallopian tubes, so fertilizes ova can be implanted into them2. She is Rh negative, a necessary stipulation to rule out Rh incompatibility

Page 42: Bullets for Nursing Basic Concepts Part 15

3. She is a normal uterus, so spem can be injected through the cervix into it

4. Her husband is taking sildenafil (Viagra), so all his sperm  will be motile

Correct Answer: 1.She has patent fallopian tubes, so fertilizes ova can be implanted into them

4.  Amy Alvarez is pregnant with her first child.  Her phenotype refers to.

1. Her concept of herself as male or female2. Whether she has 46 chromosomes or not

3. Her actual genetic composition

4. Her outward appearance

Correct Answer: 4.Her outward appearance

5. Amy Alvarez is a balanced translocation carrier for Down syndrome.  This term means that:

1. All of the children will be born with some aspects of Down syndrome2. All of her female and none of her male children will have Down syndrome

3. She has a greater than average chance a child will have Down syndrome

4. Its impossible for any of her children to be born with Down syndrome

Correct Answer: 3.She has a greater than average chance a child will have Down syndrome

6. Amy Alvarez has told a genetic councelling session that she is a balanced translocation carrier for Down syndrome.  What would be your best action regarding this information?

1. Be certain all her family members understand that this means2. Discuss the costs of various abortion techniques with Amy

3. Be sure Amy knows she hsould not have any more children

4. Ask Amy if she has any questions that you could answer for her

Correct Answer: 4. Ask Amy if she has any questions that you could answer for her

7. Amy Alvarez child is born with Down syndrome.  What is a common physical feature of newborns eith this disorder?

1. Spastic and stiff muscles2. Loose skin at back of neck

3. S white lock of forehead hair

4. Wrinkles on the sole of the feet

Page 43: Bullets for Nursing Basic Concepts Part 15

Correct Answer: 2.Loose skin at back of neck

8. Liz Calhorn asks how much longer her doctor will refer to the baby inside her as an embryo.  What would be your best explanation?

1. The tern is used during the time before fertilization.2. Her baby will be a fetus as soon as the placenta forms

3. After the 20th week of pregnancy, the baby is called a zygote

4. From the time of implantation until 5 to 8 weeks, the bay is an embryo

Correct Answer:4.From the time of implantation until 5 to 8 weeks, the bay is an embryo

1. Liz Calon is worried that her baby will be born with a congenital heart disease.  What asessment of a fetus at birth is important to help detect congenital heart defects?

9. Asessing whether the Whartons jelly of the cord has a pH higher than   7.2

1. Asessing whether the umbilical cord has 2 arteries and 1 vein2. Measuring the length of the cord to be certain that it is longer than 3 feet

3. Determining that the color of the umbilical cord is not green.

Correct Answer: 1. Asessing whether the umbilical cord has 2 arteries and 1 vein

10.   Liz calon asks you why her doctor is so concerned about whether her fetus is producing surfactant or not.  Your best answer would be:

1. Surfactant keeps lungs from collapsing on expiration, and thus newborn breathing2. Surfactant is produced by the fetal liver, so its presence reveals liver maturity

3. Surfactant is the precursor to IGM antibody production, so it prevents infection

4. Surfactant reaveals amture kidney function,as it is produced by kidney glomeruli.

Correct Answer:  1. Surfactant keeps lungs from collapsing on expiration, and thus newborn breathing

Sample Nursing Board Review Questions 2

 

1.  Liz Calon is scheduled to have an ultrasound examination.  What instruction would you give her before her examination?

 

Page 44: Bullets for Nursing Basic Concepts Part 15

1. Void immediately before the procedure to reduce bladder size2. The intravenous fluid infused to dilate your uterus does not hurt the fetus

3. You will need to drink at least 3 glasses of fluid before the procedure

4. You can have medicine for pain for any contractions caused by the test.

Correct Answer: 3.  You will need to drink at least 3 glasses of fluid before the procedure 

       2. Liz Calon is scheduled to have an amniocentesis to test for fetal maturity.  What instruction would you give her before this procedure?

 

1. void immediately before the procedure to reduce your bladder size2. The x-ray used to reveal your fetus position has no longterm effects

3. The intravenous fluid infused to dilate your uterus does not hurt the fetus

4. No more amniotic fluid forms afterward, which is why only a small amount is removed.

Correct Answer: 1.   void immediately before the procedure to reduce your bladder size 

     3. Lauren sometimes feels ambivalent about being pregnant.  What is the psychological task you’d like to see her complete during the first trimester of pregnancy?

 

1. View morning sickness as tolerable2. accept the fact that she is pregnant

3. Accept the fact the baby is growing inside her

4. Choose a name for the baby

Correct Answer: 2.    accept the fact that she is pregnant

        4. Lauren Maxwell is aware that shes been showing some narcissism since becoming pregnant.  Which of her actions best describe narcissism?

 

1. Her skin feels “pulled thin”  across her abdomen.2. Her thoughts tend to be mainly about herself

3. She feels a need to sleep a lot more than usual

4. She often feels “numb” or as if shes taken a narcotic

Page 45: Bullets for Nursing Basic Concepts Part 15

 Correct Answer: 2.  Her thoughts tend to be mainly about herself

       5. Lauren Maxwell did a urine pregnancytest but was surprised to learn that a positive result is not a sure sign of pregnancy.  She asks you what would be a positive sign.  You tell her a positive sign would be if:

 

1. she is having a consistent uterine growth2. She can feel the fetus move inside her

3. hCG can be found on her bloodstream

4. The fetal heart can be seen on ultrasound

Correct Answer: 4.  The fetal heart can be seen on ultrasound

       6. lauren Maxwell’s doctor told her she has a positive Chadwick’s sign.  She asks you what this means, and you tell her that:

 

1. Her abdomen is soft and tender2. Her uterus has tipped forward

3. Cervical mucus is clear and sticky

4. Her vagina is darkened in color

Correct Answer: 4.  Her vagina is darkened in color

        7. Lauren Maxwell overheard her doctor say that the insulin is not as effective during pregnancy as usual.  That made her worry that she is developing diabetes, like her aunt.  How would you explain how decreased insulin effectiveness safeguards the fetus?

 

1. Decreased effectiveness prevents the fetus from being hypoglycaemic2. If insulin is ineffective, it cannot cross the placenta and harm the fetus

3. The lessened action prevents the fetus from gaining too much weight

4. The mother not the fetus, is guarded by this decreased insulin action

Correct Answer: 1.   Decreased effectiveness prevents the fetus from being hypoglycaemic

        8. Sandra Czerinski feels well.  She asks you why needs to come for prenatal care.  The best reason for her to receive regular care is:

Page 46: Bullets for Nursing Basic Concepts Part 15

 

1. Discovering allergies can help eliminate early birth2. It helps document howmany pregnancies occur each year

3. It provides time for education about pregnancy and birth

4. It determines whether the pregnancy today are planned or not

Correct Answer: 3.It provides time for education about pregnancy and birth

       9. Why is it important to ask Sandra about past surgery on a pregnancy health history?

 

1. To test her recent and long term memory2. Adhesion from the surgery could limit uterine growth

3. To assess whether she could be allergic to any medication

4. To determine if she has effective health insurance

Corrrect Answer: 3.Adhesion from the surgery could limit uterine growth

 

10.  Sandra reports that the palms of her hands are always itchy.  You notice scratches on them when you do a physical exam.  What is most likely cause of this finding during pregnancy?

1. She must have become allergic to dishwashing soap2. She has an allergy on her fetus and will probably abort

3. Her weight gain has stretched the skin aver her hands

4. This is a common reaction to increasing estrogen levels

Correct Answer: 4. This is a common reaction to increasing estrogen levels

Sample Nursing Board Review Questions 3

1. Sandra has not had a pelvic exam since she was in highschool.  What advice would you give her to help her relax during her first prenatal pelvic exam?

1. have her take a deep breath and hold it during the exam

2. Tell her to bear down slightly as the speculum is inserted

3. Singing outlouds helps, because it pushes down the diaphragm

Page 47: Bullets for Nursing Basic Concepts Part 15

4. She should breathe slowly and evenly during the exam

Correct Answer: 4. She should breathe slowly and evenly during the exam

2. Sandra has pelvic measurement taken.  What size should the ischial tuberosity siameter to be to considered adequate?

1. 6cm2. Twice the width of the conjugate diameter

3. 11cm

4. Half the width of the symphysis pubis

Correct Answer: 3. 11cm

3. Which ststement by Juliberry Adams would alert you that she needs more teaching about safe practices during pregnancy?

1. “I take either a shower or tub bath, because I know  both are safe.”2. “I wash my breast with clear water, not with soap daily.”

3. “Im glad I don’t have to ask my boyfriend to use condom anymore”

4. “Im wearing low heeled shoes to try and avoid back ache”

Correct Answer:2. “Im glad I don’t have to ask my boyfriend to use condom anymore”

4. Juliberry Adams describes her typical day to you.  What would alert you that she may need further pregnancy advice?

1. “I jog rather than walk every time I can for exercise”2. “I always go to sleep on my side, not on my back”

3. “Ipack my lunch in the morning when I’m not so tired”

4. “I walk arould my desk every hour to prevent varicosities”

Correct Answer: 1.“I jog rather than walk every time I can for exercise”

5. Juliberry adams tells you she is developing oainful hemorrhoids.  Advice you would give her would be;

1. Take a tablespoon of mineral oil with each of your meal2. Omit fiber from your diet.  This will prevent constipation

3. Lie on your stomach daily to drain blood from rectal veins

Page 48: Bullets for Nursing Basic Concepts Part 15

4. Witch hazel pads feel cool against swollen hemorrhoids.

Correct Answer: 4. Witch hazel pads feel cool against swollen hemorrhoids.

6. juliberry Adams has ankle edema by the end of each day.  Which statement by her would reveal that she undertand what causes this?

1. “Iknow this is a beginning complication;  I’ll call my doctor tonight”2. “I understand this is from eating too much salt; ill restrict that more”

3. “I’ll rest in a simss position to take pressure off lower extremity veins”

4. “I walk for half an hour everyday to relieve this; I’ll try walking more”

Correct Answer: 3. “I’ll rest in a simss position to take pressure off lower extremity veins”

7. Julliberry Adams makes the following statements.  Which one is the safest practice?

1. “My brother takes medicine for heartburn; if I get that ill just borrow his”2. “Im going to get a measles shot; I don’t want measles while I am pregnant”

3. “there are so many medicines for headache, I have to ask my doctor what to take”

4. “I know all-over-the-counter medicine is safe; it’s why it’s over all counter.”

Correct Answer: 3. “there are so many medicines for headache, I have to ask my doctor what to take”

8. Suppose Toni Alarino has a normal BMI.  What would be the recommend weight gain for her during pregnancy?

1. 10lbs2. 20lbs

3. 30lbs

4. 60lbs

Correct Answer: 3. 30lbs

9. Which statement by Toni lets you know she understand how iron is best absorbed

1. I always take my iron pills with milk2. I take iron pills mixed with mashed potatoes

3. I take the pills with ice cream to disguise the taste

4. I swallow my iron pills with orange juice

Page 49: Bullets for Nursing Basic Concepts Part 15

Correct Answer: 4. I swallow my iron pills with orange juice

10.  You need to obtain a nutrition history from Toni.  What is the best way to do this?

1. Ask her to tell you how much protein she eats daily2. Asess if Toni feels satisfied or not with her nutrition

3. Ask Toni to describe what she ate in the last 24 hours

4. tell Toni to describe her concept of ideal nutrition

Correct Answer: 4. Ask Toni to describe what she ate in the last 24 hours

Sample Nursing Board Exam Review Questions 4

 

1. An infant is born with a bilateral cleft palate. Plans are made to begin reconstruction immediately. Nursing intervention to promote parent-infant bonding should include:

 

A. Demonstrating a positive acceptance of the infant.B. Placing the baby in a nursery away from view of the general public.C. Explaining to the parents that the infant will look normal after the surgery.D. Encouraging the parents to limit contact with the infant until after surgery.

Correct Answer: ARationale: A. By demonstrating acceptance of the infant, without regard for the defect, the nurse acts as a role model for the parents, thus enhancing their acceptance. B. Infants with cleft palates can remain in the newborn nursery; they should not be hidden. C. This is false reassurance; it does not promote parent-infant bonding.D. The parents should be encouraged to have frequent contact with their infant to promote bonding.

 

2. A pregnant client’s labor is to be induced at 39 week’s gestation. The nurse is aware that several drugs are currently utilized for inducing labor. Select all that apply.

 

A. Oxytocin (Pitocin)B. Misprostol (Cytotec)C. Ergonovine (Ergotrate)

Page 50: Bullets for Nursing Basic Concepts Part 15

D. Carboprost (Hemabate)E. Dinoprostone (Prepidil)

Correct Answer:: ARationale: Pitocin is an oxytocic used for labor induction. Cytotec is a prostaglandin used for cervical ripening and labor induction. Ergotrate is an oxytocis used for postpartum or postabortion hemorrhage, not labor induction. Hemobate is a prostaglandin used for pospartum hemmorrhage, not labor induction. Prepidil is used for cervical ripening, not labor induction.

 

3. It is important for the nurse to support the parent’s decision to abort a fetus with a birth defect because:

 

A. Supporting them will eliminate feelings of guiltB. The parents are legally responsible for the decisionC. It is essential for maintenance of family equilibriumD. The nurse’s support will relieve the pressure associated with decision making

Correct Answer: CRationale: A. Support may help, but it does not completely alleviate guilt feelings. B. Support does not affect the legal responsibility of the parents. C. Although support will help minimize guilt, it will not eliminate it; however will sustain family cohesion and unity. D. This may help, but it cannot completely relieve pressure.

 

4. During the first hours following delivery, the postpartum client is given intravenous fluids with oxytocin (Pitocin) added to them. The nurse understands the primary reason for this is:

 

A. To facilitate eliminationB. To prevent infectionC. To promote analgesiaD. To promote uterine contraction

Correct Answer: DRationale: Oxytocin is a hormone produced by the pituitary gland that produces intermittent uterine contractions, helping to promote uterine involution. The intravenous fluid may facilitate elimination, but oxytocin does not affect bowel or bladder elimination. Oxytocin has no antibacterial or analgesic action.

 

Page 51: Bullets for Nursing Basic Concepts Part 15

5. During the postpartum period, while considering nursing measures to help parent-child relationships, the nurse should be aware that the most important factor at this time is the:

 

A. Anesthesia during laborB. Duration and difficulty of laborC. Physical condition of the infantD. Health status during pregnancy

Correct Answer: CRationale: A. Though the effect of an anesthesia is a factor, the most important factor is the physical condition of the infant.B. Though the duration and difficulty of labor is a factor, the most important factor is the physical condition of the infant. C. Bonding between parent and baby is most successful when interaction is possible right after birth; if the child is ill, contact is limited. D. Health status during pregnancy may be a factor, but the most important factor is the physical condition of the infant.

 

6. The uterus rise out of the pelvis and becomes an abdominal organ at about the:

 

A. 10th week of pregnancyB. 8th week of pregnancyC. 12th week of pregnancyD. 18th week of pregnancy

Correct Answer: CRationale: A. The uterus is still within the pelvic area. B. The uterus is still within the pelvic area. C. By this time the fetus and placenta have grown, expanding the size of the uterus. The extended uterus expands into the abdominal cavity. D. The uterus has already risen out of the pelvis and is expanding farther into the abdominal area.

 

7. A client suspects that she is pregnant, but because she is the only wage earner in her family, she is ambivalent about continuing the pregnancy. The nurse recognizes that the client is in crisis and also remembers that pregnancy and birth are considered crises because:

 

A. There are mood changes during pregnancyB. They are periods of change and adjustment to change

Page 52: Bullets for Nursing Basic Concepts Part 15

C. There are hormonal and physiologic changes in the motherD. Narcissism is the mother affects the husband wife relationship

 Correct Answer: BRationale: A. These are transient; they are similar to previous mood changes and should not affect the mother’s ability to cope. B. Expected periods of marked change and adjustment are called developmental crises and predispose the woman to a situational crisis. C. These occur throughout the life cycle of a mature woman and should not now be classified as a crisis. D. It becomes a crisis only if the husband withdraws support.

 

8. When caring for a family on a postpartum unit, the nurse must be aware that all the tasks, responsibilities, and attitudes that make up child care can be called parenting and that either parent can exhibit these qualities. A person is able to perform parenting because of:

 

A. A marriage with flexible rolesB. An inborn ability based on instinctC. Positive childhood roles and conceptsD. A good education in growth and development

Correct Answer: CRationale: A. Marriage is not essential for good parenting. B. Parenting is learned, not inborn. C. Parenting is not an inborn instinct rather a learned behavior based on past experiences or current instruction. D. This knowledge does not ensure the ability to parent.

 

9. During labor a client who has been receiving epidural anesthesia has a sudden episode of severe nausea, and her skin becomes pale and clammy. The nurse’s immediate reaction should be to:

 

A. Notify the physicianB. Elevate the client’s legsC. Check for vaginal bleedingD. Monitor the FHR every 3 minutes

Correct Answer: BRationale: A. If signs and symptoms do not abate after elevation of the legs, the physician should be notified. B. Maternal hypotension is a common complication of this anesthesia for labor, and nausea is one of the first clues that this has occurred. Elevating the extremities restores blood to the central circulation. C. This is not a specific observation after caudal anesthesia; it is part of

Page 53: Bullets for Nursing Basic Concepts Part 15

the general nursing care during labor. D. If the FHR is being monitored, it is a constant process; if not, the FHR should be monitored every 15 minutes.

 

10. After ovulation has occurred, the ovum is believed to remain viable for:

 

A. 1 to 6 hoursB. 12 to 18 hoursC. 24 to 36 hoursD. 48 to 72 hours

 Correct Answer: CRationale: A. The ovum is viable a longer time. B. The ovum is viable a longer time. C. The ovum is capable of being fertilized for only 24 to 36 hours following ovulation; after this time it travels a variable distance between the fallopian tube and uterus, disintegrates, and is phagocytized by leukocytes. 4. The ovum is not fertilizable after 36 hours.

Sample Nursing Board Exam Review Questions 5

1. Supportive nursing care in the beginning mother-infant relationship should include:

A. Requiring the mother to assist with simple aspects of her infant’s careB. Encouraging the mother to decide between breastfeeding and formula feedingC. Allowing the mother ample time to undress and to carefully inspect her infantD. Unobtrusive observation of the mother and her infant to watch for a disturbed relationship

Correct Answer: CRationale: A. The client will proceed at her own rate; requiring her to do things is not supportive. B. The mother should have made this decision before delivery. C. Allowing the mother time to inspect the child permits viewing, touching, and holding, promoting bonding. D. This can be done only by allowing the mother ample time to interact with her baby.

2. A client undergoing treatment for infertility is diagnosed as having endometriosis. The nurse is aware that one of the drugs that may be used to treat this condition is:

A. Relaxin (Releasin)B. Leuprolide (Lupron)C. Ergonovine (Ergotrate)D. Esterfied estrogen (Climestrone)

Correct Answer: BRationale: Continuous administration of Lupron decreases LH and FSH, as well as hormone-dependent tissue. Relaxin is used for dysmenorrhea; it causes relaxation of the symphysis pubis.

Page 54: Bullets for Nursing Basic Concepts Part 15

Ergotrate is used to contract the uterus. Esterfied estrogen (Climestrone) is an estrogen that affects release of pituitary gonadotropins and inhibits ovulation.

3. Research concerning the emotional factors of pregnancy indicates:

A. A rejected pregnancy will result in a rejeted infantB. Ambivalence and anxiety about mothering are commonC. Maternal love is fully developed within the first week after birthD. An effective mother experiences neither ambivalence nor anxiety about mothering

Correct Answer: BRationale: A. Frequently the maternal instinct is nurtured by the sight of the infant. B. Because mothering is not an inborn instinct, almost all mothers, including multiparas, report some ambivalence and anxiety about their ability to be good mothers. C. It may take a much longer time. D. Ambivalent feelings are universal in response to a neonate.

4. Which of the following instructions would be included in a client’s postpartum teaching plan about performing Kegel exercises to restore perineal muscle tone?

A. Alternately flexing and extending each foot while raising her leg 6 inches off the bedB. Contracting and relaxing perineal muscles as if stopping and starting a urinary streamC. Contracting the abdominal muscles while raising her legs 1 inch off the bedD. Taking deep breaths and slowly exhaling while compressing the buttocks together

Correct Answer: BRationale: Kegel exercises require contracting the pubococcygeal muscle, the major muscle of the pelvic floor, to increase muscle tone and provide support to the vaginal wall, bladder, and rectum. Alternate flexing and extending the foot with raising will facilitate venous return and help prevent thrombophlebitis. However, this exercise will not affect perineal muscle tone. Contracting the abdominal muscles with leg raises will increase abdominal, but not perineal, muscle tone. Deep breathing expands the lungs. Contracting gluteal muscles will not affect the perineal, muscle tone.

5. A client visiting the prenatal clinic for the first time asks the nurse about the probability of having twins because her husband is one of a pair of fraternal twins. The nurse should explain that:

A. The probability of having twins is 25%B. She will be monitored closely for the presence of twinsC. Her husband’s history of being a twin raises the probability of having twinsD. There is no greater probability of having twins than in the general population

Correct Answer: DRationale: A. If there is no maternal family history of twin pregnancies, it would be a chance occurrence that is equal to the probability found in the general population. B. Pregnant women are routinely monitored for multiple pregnancies; this client needs information about her risk for

Page 55: Bullets for Nursing Basic Concepts Part 15

having twins. C. If there is no maternal family history of twin pregnancies, it would be a chance occurrence that is equal to the probability found in the general population. D. Fraternal twins may occur as a result of a hereditary trait, but is related to the ovaries releasing two eggs during one ovulation; the fact that the father is a fraternal twin would not influence the female to release two eggs during one ovulation.

6. During the process of gametogenesis, the male and female sex cells divide, and each mature sex cell contains:

A. 24 pairs of autosomes in their nucleiB. 46 pairs of chromosomes in their nucleiC. A diploid number of chromosomes in their nucleiD. A haploid number of chromosomes in their nuclei

Correct Answer: DRationale: A. They each have one set of chromosomes (23). B. There are only 23 pairs of chromosomes in the nuclei. C. The diploid number (46 chromosomes) is reached when fertilization occurs. D. This is the result of a reduced chromosome number, from 46 to 23, readying the sex cells for fertilizaiton.

7. During the postpartum period, a cardiac client with type 2 diabetes asks the nurse, “Which contraceptives will I be able to use to prevent pregnancy in the near future?” The nurse’s best response would be:

A. “You may use oral contraceptives. They are almost 100% effective in preventing pregnancy.”B. “You may want to use a foam and a condom to prevent pregnancy until you consult with your doctor at your postpartum visit.”C. “The intrauterine device is best for you because it does not allow a fertilized ovum to become implanted in the urerine lining.”D. “You do not need to worry about becoming pregnant in the near future. Clients with cardiac conditions usually become infertile.”

Correct Answer: BRationale: A. Oral contraceptives are not recommended for this client because of their tendency to alter glucose tolerance. B. Some type of a barrier contraceptive (condom) is usually recommended for the client with diabetes mellitus and a cardiac condition. C. An IUD is not recommended because it may predispose this client to infection. D. This is untrue; clients with a cardiac condition can become pregnant again in the future.

8. A client at 6 weeks’ gestation is receiving antibiotic theraphy for pyelonephritis. The nurse is aware that the safest antibitioc for administration during pregnancy is:

A. GantrisinB. AmpicillinB. TetracyclineD. Nitrofurantoin

Page 56: Bullets for Nursing Basic Concepts Part 15

Correct Answer: BRationale: Ampicillin has no know tertogenic effect associated with penicillin. Gantrisin sulfonamides may cause hemolysis in the fetus. Tetracycline causes permanent yellow staining of teeth in children whose mothers receive the drug during pregnancy. Nitrofurantion is contraindicated in severe renal disease.

9. A client with multiple sclerosis has just confirmed her pregnancy. She states she is taking ACTH and wonders whether she can continue taking it. The best response by the nurse would be:

A. ACTH is safe to take during pregnancy

B. ACTH may make you prone to nausea and vomitingC. This medication is untested in pregnancy. Check with your physician.D. This is used during plasmapheresis and should be safe to continue taking.

Correct Answer: ARationale: Although ACTH is a pregnancy category C drug and it is not known whether it is harmful to the fetus, the client’s health must be considered as well as the life of the fetus; it acts to strengthen nerve conduction. Nausea and vomiting are not side effects of ACTH. ACTH has been tested in pregnancy. Used during plasmapheresis is not the reason ACTH has been prescribed; the client has multiple sclerosis.

10. After the first 3 months of pregnancy, the chief source of estrogen and progesterone is the:

A. PlacentaB. Adrenal CortexC. Corpus luteumD. Anterior hypophysis

Correct Answer: ARationale: A. When placental formation is complete, around the 12th week of pregnancy, it produces progesterone and estrogen. B. This is not the chief source of progesterone and estrogen; only small amounts are secreted. C. The corpus luteum supplies the estrogen and progesterone needed to sustain the pregnancy until the placenta is ready to take over. D. FSH is secreted by the anterior hypophysis, but it is not secreted during pregnancy.

Sample Nursing Board Exam Review Questions 6

1. A young couple attends the prenatal clinic. The wife is 8 weeks’ pregnant and asks the clinic nurse for information about an abortion. The nurse expresses the opinion that abortion is immoral and that many women have long-term guilt feelings after an abortion. The couple leave the clinic in a very disturbed state. Legally, the:

Page 57: Bullets for Nursing Basic Concepts Part 15

A. Client had a right to receive correct, unbiased informationB. Nurse’s statements need not be based on scientific knowledge

C. Physician should have been called in, since nurses should not discuss abortionD. Nurse had a right to state feelings as long as they were identified as the nurse’s own

Correct Answer: ARationale: A. Nurses with positive attitudes toward abortion should counsel women who are thinking of undergoing the procedure; they should know what services are available and the various methods that are used to induce abortion. B. Nursing practice necessitates scientific knowledge; statements must be based on fact, not personal feelings or beliefs. C. The nurse is capable of giving information about abortion and need not defer to the physician. D. The nurse should give the client only the information requested and should not state personal feelings.

2. A newly delivered mother with three young children at home comment to the nursery nurse that she cannot hold the baby for feedings once she gets home. She has just too much to do, and anyhow, it spoils the baby. The best response for the nurse to make is:

A. “You seem concerned about time. Let’s talk about it.”B. “That’s entirely up to you; you have to do what works for you.”C. “Holding the baby when feeding is important for development.”D. “It is very unsafe to prop a bottle. The baby could aspirate the fluid.”

Correct Answer: ARationale: A. This opens up an area of communication to get at what really is troubling the mother about feeding the baby. B. Because the nurse is aware that this is not the best method, the problem of time should be explored with the mother. C. Holding can be accomplished at times other than feeding periods; it does not explore the client’s feelings. D. This is true, but the mother should not be frightened; a more gentle explanation should be used.

3. Which of the following is the most important nursing action when caring for a client who is 6 hours post cesarean birth?

A. Turning, coughing, and deep breathing every 4 hoursB. Applying anesthetic spray to her perineumC. Assisting the client to the bathroom for a showerD. Encouraging a sitz bath three times a day

Correct Answer: ARationale: A cesarean birth is a surgical procedure involving an abdominal incision. Immobility in the postoperative period causes secretions to pool. Also, abdominal incisions cause pain on deep breathing. Clients with abdominal incisions need to turn, cough, and deep breathe to mobilize secretions to prevent atelectasis and pneumonia. Anesthetic spray is applied to the episiotomy to provide local analgesia. Clients who have had a cesarean birth do not have a perineal incision. Six hours post cesarean birth is too soon for a shower. The client may be very

Page 58: Bullets for Nursing Basic Concepts Part 15

weak for the first 24 hours. Safety might be an issue. Sitz baths provide cleansing and warmth to perineal areas for clients with episiotomies.

4. The husband of a client who is in the transitional phase of labor becomes very tense and nervous during this period and asks the nurse, “Do you think it is best for me to leave, since I don’t seem to do my wife much good?” The most appropriate response by the nurse would be:

A. “This is the time your wife needs you. Don’t run out on her now.”B. “This is hard for you. Let me try to help you coach her during this difficult phase.”C. “I know this is hard for you. Why don’t you go have a cup of coffee and relax and come back later if you feel like it?”D. “If you feel that way, you’d bes go out and sit in the father’s waitingroom for a while because you may transmit your anxiety to your wife.”

Correct Answer: BRationale: A. This statement is judgmental; this approach suggest that he will be failing his wife. B. Both the father and the mother need additional support during the transitional stage of labor. C. The husband should be present throughout labor to support his wife; he should be assisted in this role. D. This does not encourage him to fulfill his role in supporting the mother during labor.

5. A decision to withhold “extraordinary care” for a newborn with severe abnormalities is actually:

A. A decision to let the newborn dieB. The same as pediatric euthanasiaC. Presuming that the newborn has no rightsD. Unethical and illegal medical nursing practiceCorrect Answer: ARationale: A. Based on the family’s decision, extraordinary care does not have to be employed; the infant’s basic needs are met, and nature is allowed to take its course.B. Euthanasia is a deliberate intervention to cause death. C. If the infant’s physical needs are met and comfort is provided, the infant’s rights are not ignored; “extraordinary,” not “all,” care is being withheld. D. It is neither unethical or illegal to withhold extraordinary treatments; once such treatment is started, it becomes a legal issue.

6. In dealing with a couple identified as having an infertility problem, the nurse knows that:

A. Infertility is usually psycholigic in originB. Infertility and sterilty are essentially the same problemC. The couple have been unable to have a child after trying for a yearD. One partner has a problem that makes that person unable to have children

Correct Answer: CRationale: A. Infertility may be psychogenic; however, statistics show that physiologic problems are more often the cause. B. This is untrue; infertility may be corrected, but sterility is

Page 59: Bullets for Nursing Basic Concepts Part 15

irreversible. C. Infertility is the inability of a couple to conceive after at least 1 year of adequate exposure to the possibility of pregnancy. D. This may or may not be true; it is possible that there is a problem with both.

7. Which of the following assessment findings about the uterus would the nurse expect to find in a primipara client 6 hours post delivery of an average-for-gestational-age infant?

A. Fundus firm, midline, 2 fingerbreadths below the umbilicusB. Fundus firm, midline, at the level of the umbilicusC. Fundus firm, to the right of the midline, at the umbilical levelD. Fundus soft, to the right of the midline, 2 fingerbreadths above the umbilicus

Correct Answer: BRationale: One hour after birth, the fundus rises to the level of the umbilicus, where it remains for approximately 24 hours. It should be firm and in the middle. The fundus should be firm and midline, but should not be 2 fingerbreadths below the umbilicus on the day of delivery. A fundus that is to the right of the midline denotes urinary bladder distention. The fundus should be firm to provide hemostasis. The fundus is up too far in the abdomen and is deviated to the right, denoting a distended bladder.

8. An amniocentesis done on a client at 16 weeks’ gestation reveals a fetus with Down syndrome. The client and her husband elect to have the pregnancy terminated. The nurse giving care to a client whose pregnancy is surgically terminated should be aware that:

A. The client is emotionally unstable at this timeB. There is a high risk for a postoperative infectionC. Contraceptive counseling should be deferred to a later timeD. The client needs to express her feeling of guilt, anger, and frustration

Correct Answer: DRationale: A. This is a false assumption. B. This is a sterile procedure and should not predispose the client to postoperative infection. C. Studies show that contraceptive counseling at this time is most important, because the client may not return after the abortion. D. The client must feel comfortable enough to verbalize her feelings of guilt; this helps to complete the grieving process.

9. A couple in the fertility clinic have become very discouraged regarding their efforts to conceive. The nurse can best support them by understanding that the most stressful aspect of the process is:A. Obtaining the necessary specimensB. Visitng the fertility clinic frequentlyC. Discovering which partner is infertileD. Planning when intercourse should take place

Correct Answer: D Explanation: A. Obtaining and delivering the necessary specimens may be inconvenient but should not be stressful. B. The number of office visits and examinations that are required may be cumbersome but should not be stressful. C. The couple probably knows that one

Page 60: Bullets for Nursing Basic Concepts Part 15

of them has a fertility problem; it may be helpful knowing that the problem is so that measures can be taken to correct it. D. A strategy for increasing the chances of conceiving requires the couple to plan intercourse only while the woman is ovulating; this removes spontaneity and is often stressful.

10. During the taking-hold phase, the nurse would expect the new mother to:

A. Talk about the babyB. Call the baby by nameC. Touch the baby with her fingertipsD. Be passively involved with the babyCorrect Answer: BRationale: A. This may occur in either phase. B. The mother has completed the taking-in phase (the mother’s needs predominate) and has moved into taking-hold the taking-hold phase (active maternal involvement with self and infant) when she calls the baby by name. C. This is the initial early action of the taking-in phase. D. This is part of the taking-in phase.

Sample Nursing Board Exam Review Questions 7

1. The nurse recognizes that an expected change in the hematologic system that occurs during the second trimester of pregnancy is:

A. A decrease in WBCsB. An icrease in blood volumeC. An increase in blood volumeD. A decrease in sedimanation rate

Correct Answer: CRationale: A. White blood cell values remain stable during the antepartum period. B. The hematocrit decreases as a result of hemodilution. C. The blood volume increases by approximately 50% during pregnancy. Peak blood volume occurs between 30 and 34 weeks of gestation. D. The sedimentation rate increases because of a decrease in plasma proteins.

2. In the 12th week of gestation, a client completely expels the products of conception. Because the client is Rh-negative, the nurse must:

A. Administer RhoGAM within 72 hoursB. Make certain she recieves RhoGAM on her first clinic visitC. Not give RhoGAM, since it is not used with the birth of a stillbornD. Make certain the client does not receive RhoGAM, since the gestation lasted only 12 weeks

Correct Answer: ARationale: A. It is given within 72 hours postpartum if the client has not been sensitized previously. B. It would be useless at this time. C. RhoGAM is always indicated at the termination

Page 61: Bullets for Nursing Basic Concepts Part 15

of a pregnancy, even with fetal demise. D. RhoGAM is always indicated at the termination of a pregnancy, even with a short-term pregnancy.

3. During prenatal development, fetal weight gain is greatest in the:

A. First TrimesterB. Third TrimesterC. Second Trimester

D. Implantation PeriodCorrect  Answer: BRationale: A. The first trimester is the period of organogenesis, when cells differentiate into major organ systems. B. This is the period in which the fetus sores deposits of fat fetal weight gain is greatest. C. Growth is occurring, but fat deposition does not occur in this period. D. This is the period of the blastocyst, when initial cell division takes place.

4. A client at 38 weeks’ gestation is admitted for induction of labor. She has a history of ruptured membranes for the past 12 hours. She has no other symptoms of labor. The nurse is aware that if the proper conditions exist, the physician will prescribe:

A. ProgesteroneB. Oxytocin (Pitocin)C. Lututrin (Lutrexin)D. Ergonovine maleate

Correct Answer: BRationale: Oxytocin is a small polypeptide hormone normally synthesized in the hypothalamus and secreted from the neurohypophysis during parturition or suckling; the synthetic form promotes powerful uterine (smooth muscle) contracitons and thus is used to induce labor. Progesterone builds up the endometrium; it does not initiate uterine contractions. Lututrin no drug by this name for this purpose. Ergonovine can lead to sustained contractions, which would be undesirable labor.

5. A client, whose husband is overseas in the military, is admitted to the hospital with vaginal staining but no pain. The client’s history reveals amenorrhea for the last 2 months and pregnancy confirmation by her physician after her first missed period. She is admitted for observation with a possible diagnosis of :

A.Missed abortionB. Ectopic pregnancyC. Inevitable abortionD. Threatened abortion

Correct Answer: DRationale: A. This may not cause any outward symptoms, only the signs of pregnancy disappearing. B. This is usually accompanied by severe pain radiating to the shoulder on the

Page 62: Bullets for Nursing Basic Concepts Part 15

affected side. C. This can be confirmed only if vaginal examination reveals cervical dilation. D. Spotting in the firs trimester may indicate that the client may be having a threatened abortion; any client with the possibility of hemorrhage should not be left alone; therefore admitting this client for observation is safe medical practice; abortion is usually inevitable if accompanied by pain and cervical dilation.

6. At about 5 cm dilation, a laboring client receives medication for pain. The nurse is aware that one of the medications given to women in labor that could cause respiratory depression of the newborn is:

A. ScopolamineB. Promazine (Sparine)C. Meperidine (Demerol)D. Promethazine (Phenergan)

Correct Answer: CRationale: Respiratory depression occurs with the use of meperidine (Demerol) and produces significant depression of the infant at birth if circulating levels are high at time of birth. Scopolamine induces amnesia and forgetfulness in the mother but does not cause respiratory depression; this medication is not presently used. Prpmazine (Sparine), an anxiolytic, augments the effects of demerol, thereby lessening the amount of drug needed. Promethazine (Phenergan), an antihistamine, does not cause respiratory depression.

7. When caring for a client who is having a prolonged labor, the nurse must be aware that the client is very concerned when her labor deviates from what she sees as the norm. A response conveying acceptance of the client’s expressions of frustration and hostility would be:

A. “I’ll rub your back; tell me if it helps.”B. “I’ll leave as you can talk to your husband.”C. “All women get weary and frustrated during labor.”D. “Would you like to talk about what’s bothering you?”

Correct Answer: ARationale: A. This response provides the client with a comfort measure while giving her an opportunity to get verbalize her fears about having a prolonged labor. B. This closes off communication with the client. C. This is of no help to the client; she is concerned with what is happening to her. D. This can be answered “yes” or “no” and leaves no further avenue for discussion.

8. A pregnant woman is at term is admitted to the birthing unit in active labor. The client is excited about the anticipated birth because has three sons and the amniocentesis indicates that she will have a girl. The nurse recognizes that there are implications for newborn observations and care when the nursing history reveals that:

A.The membranes ruptured 2 hours agoB. Her first child was diagnozed with hemophilia

Page 63: Bullets for Nursing Basic Concepts Part 15

C. She has taken NSAIDs for frequent sinus headachesD. There was a placenta previa in a previous pregnancy

Correct Answer: CRationale: NSAID as well as other over-the counter drugs (OCT) taken during pregnancy may cause problems in the newborn during the neonatal period. The membranes ruptured 2 hours ago is not a cause of concern; if membranes ruptured over 24 hours before birth, infection may ensue. Hemophilia affects males; this fetus is known to be a female may be a carrier but would not have hemophilia. Placenta previa would have been diagnosed before active labor; a history of a placenta previa in an earlier pregnancy would not have implication for this newborn.

9. A 16-year-old comes to the prenatal clinic because she has missed three menstrual periods. Before her physical examination, the client says. “I don’t know what the problem is, but I can’t be pregnant.” The nurse’s most therapeutic response to this statement would be:

A. “The doctor will let you know shortly.”B. “What brought you to the prenatal clinic then?”C. “Many young women are irregular at your age.”D. “If you have had intercourse, you are probably pregnant.”

Correct Answer: BRationale: A. This response would close off any future communication with the client. B. This response points out reality and allow the client to elaborate.C. This may be true statement, but it does not allow for much discussion to follow. D. This response sounds rather critical or judgmental and would probably cut off further discussion with the client.

10. After an 8-hour, uneventful labor a client gives birth to a baby boy spontaneously under epidural block anesthesia. As the nurse places the baby in the mother’s arms immediately after the birth, the mother asks, “is he normal?” The most appropriate response by the nurse would be:

A. “Most babies are normal; of course he is”B. “He must be all right, he has such a good strong cry.”C. “Yes, because your pregnancy and labor were so normal.”D. “Shall we unwrap him so you can look him over for yourself?”

Correct Answer: DRationale: A. This is false reassurance; this comment closes off communication with the mother at a very opportune moment. B. Crying is not indicative of congenital defects; a strong cry does not ensure “normalcy”. C. The “normalcy” of the mother’s pregnancy and labor does not always have a relationship to the “normalcy” of the infant. D. Mothers need to explore their infants visually and through touch to assure themselves that the infants are normal in all respects.